Вы находитесь на странице: 1из 65

Criminal Law Review- Midterm Consolidated Cases there is no doubt that Petrus is liable as principal of the crime of

kidnapping for ransom. Susana, on the other hand, is liable only as an


accomplice to the crime as correctly found by the lower courts. It must be
emphasized that there was no evidence indubitably proving that Susana
PEOPLE v PETRUS YAU AND SUSANA YAU y SUMOGBA; G.R. No. participated in the decision to commit the criminal act.
208170; August 20, 2014
Jurisprudence is instructive of the elements required, in accordance with
Article 18 of the RPC, in order that a person may be considered an
accomplice, namely;
I. The elements of Kidnapping For Ransom under Article 267 of the
RPC, as amended by R.A. No. 7659, are as follows: (1) that there be a community of design; that is, knowing the
criminal design of the principal by direct participation, he
(a) intent on the part of the accused to deprive the victim of his concurs with the latter in his purpose;
liberty;
(2) that he cooperates in the execution by previous or
(b) actual deprivation of the victim of his liberty; and simultaneous act, with the intention of supplying material or
moral aid in the execution of the crime in an efficacious
(c) motive of the accused, which is extorting ransom for the release way; and
of the victim.
(3) that there be a relation between the acts done by the
All of the foregoing elements were duly established by the principal and those attributed to the person charged as
testimonial and documentary evidences for the prosecution in the case at accomplice.
bench. First, Petrus is a private individual. Second, Petrus kidnapped
Alastair by using sleeping substance which rendered the latter unconscious In the case at bench, Susana knew of the criminal design of her husband,
while inside a taxicab driven by the said accused-appellant. Third, Petrus Petrus, but she kept quiet and never reported the incident to the police
took and detained Alastair inside the house owned by him and Susana Yau authorities. Instead, she stayed with Petrus inside the house and gave food
in Bacoor, Cavite, where said victim was handcuffed and chained, and to the victim or accompanied her husband when he brought food to the
hence, deprived of his liberty. Fourth, Alastair was taken against his will. victim. Susana not only countenanced Petrus illegal act, but also supplied
And fifth, Petrus made demands for the delivery of a ransom in the amount him with material and moral aid. It has been held that being present and
of US$600,000.00 for the release of the victim. giving moral support when a crime is being committed make a person
responsible as an accomplice in the crime committed. As keenly observed
II. Criminal liability of each accused-appellant; As a Principal; As an by the RTC, the act of giving food by Susana to the victim was not essential
Accomplice and indispensable for the perpetration of the crime of kidnapping for
ransom but merely an expression of sympathy or feeling of support to her
husband. Moreover, this Court is guided by the ruling in People v. De
Vera, where it was stressed that in case of doubt, the participation of the
offender will be considered as that of an accomplice rather than that of a driver was asked to scroll down his window and was told that the
principal. vehicle was being used to victimize foreign nationals. Appellant
did not offer to make any comment. Hence, this prompted the
III. Penalty officers to ask for his name and since he answered that he was
Petrus Yau, a British national, they asked him for his drivers
With respect to the penalty, the Court finds that the RTC was correct in license and car registration but appellant was not able to produce
imposing the penalty of reclusion perpetua without eligibility of parole any. Since he could not produce any drivers license and car
against Petrus as principal in the charge of kidnapping for ransom in view registration, they were supposed to bring him to the police station
of R.A. No. 9346, prohibiting the death penalty. Also, the Court finds that for investigation, however, when shown a picture of private
the penalty of eight (8) years and one (1) day of prision mayor, as complainant and asked if he knew him, he answered that the man
minimum, to twelve (12) years and ten (10) months of reclusion is being kept in his house. He was immediately informed that he
temporal, as maximum, meted out against Susana, an accomplice, to be was being placed under arrest for kidnapping private complainant
proper. Alastair Onglingswam after being informed of his constitutional
rights. rescue operations of the victim proceeded thereafter;
The entire amount of the civil liabilities should be apportioned among all
those who cooperated in the commission of the crime according to the 4. When the police proceeded inside the house they found a man
degrees of their liability, respective responsibilities and actual participation. sitting on the floor chained and handcuffed. The man later
Accordingly, Petrus should shoulder a greater share in the total amount of identified himself as Alastair Onglingswam;
damages than Susana who was adjudged only as an accomplice. (People v.
Montesclaros) 5. RTC finds them guilty beyond reasonable doubt and rejected the
twin defenses of alibi and frame-up submitted by Petrus and
FACTS: Susana because the same were unsubstantiated by clear and
convincing evidence
1. Petrus and Susana were charged with the crime of Kidnapping For
Ransom of private complainant Alastair Onglingswam, who was 6. The CA affirmed the conviction of Petrus and Susana, Hence this
put into sleep while riding a taxi (White Toyota taxi cab with plate assignment of errors.
number PVD-115) bound to San Juan, Manila. Subsequently, Chau
(Alastair girlfriend) received an email from the purported
kidnapper demanding US$2,000.00;
ISSUES
2. During twenty-two (22) days of captivity, he was allowed to
communicate with his family almost daily to prove that he was still The focus of this case is the degree of responsibility of each
alive and was served with meals almost five times a day either by accused-appellant for the crime of kidnapping for ransom.
Petrus or the other accused Susan Yau;
Case no. 2
3. The police were able to chance upon the said vehicle. they People of the Philippines v. Gonzales
followed it, then flagged it down and approached the driver. The 183 SSCRA 309, March 19, 1990
-Sixteen wounds: five fatal as they penetrated the
Doctrine: The commission of a felony under Art. 3 of the Revised Penal internal organs
Code requires that an act a punishable act or omission must be committed, -Multiple puncture, stab, incision, and lacerated
and that it must be committed with deceit and/or fault. wounds
-The day after the autopsy, Augusto appeared before the sub-
station and voluntarily surrendered to Police Corporal Sazon for
FACTS: detention and protective custody for having been involved in the
killing of the deceased. Augusto requests to be taken to where
The appellant is appealing to the court regarding his participation Fausta was already detained.
in the killing of a certain Loly Penacerrada. He claims that he did not
participate in the killing based on the claim that he was not present in the Based on the investigations conducted, an information for murder
said act. dated August 26, 1981, was filed by the Provincial Fiscal of Iloilo against
the spouses. However, they pleaded not guilty. Before the trial, however, a
The antecedent facts are as follows: certain Jose Huntoria presented himself to the wife of the deceased.
Huntoria claims to be a witness of the killing, and on October 6, 1981,
At around 9:00 p.m. of February 21, 1981, Bartolome Paja, volunteers as a witness for the prosecution. A reinvestigation of the case
barangay captain of Brgy. Tipacla, Ajuy, Iloilo, was awakened by was called, in which several more were filed as accused, including the
two of the accused. Paja learns that Fausta killed their landlord, appellant. All the accused except for Lenida pleaded not guilty
Lloyd Penacerrada, and would like to surrender to authorities.
Knife used in killing was seen, and blood was found smeared on At the trial, the prosecution presented Dr. Jesus Rojas, the
Faustas dress. physician who conducted the autopsy on the body, Paja, the patrolmen and
constabulary members who joined in the investigation, the widow, and
-Paja immediately ordered a nephew to take spouses to the police Huntoria.
at the Municipal Hall in Poblacon, Ajay, where the couple
informed the police on duty of the incident. Dr. Rojas testified that he performed the autopsy at around 11:20
a.m. on Feb. 1981 after the deceased was taken to the municipal hall. He
-Several patrolmen, along with Paja and Augusto proceeded to the found 4 puncture wounds, 7 stab wounds, 4 incisions, and1 laceration; five
residence at Sitio Nabitasan where the killing incident allegedly of these were fatal wounds. Rojas admitted one of two possibilities:-
occurred, and found the body of the deceased, clad in underwear,
sprawled face down inside the bedroom. -Only one weapon might have caused all the wounds-
-Multiple instruments were used due to the number and
-Group stayed for an hour in which the scene was inspected, and a different characteristics
rough sketch of the area was made.

-The next day, a patrolman, accompanied by a photographer, went The substance of the prosecutions case rested on Huntorias
back to the scene for further investigations. Fausta was brought alleged eyewitness account of theincident, which was as follows:
back to the police station.
-Testified on July 27, 1982; at 5 pm on Feb. 21, 1981, he
-The autopsy of the deceased was performed at 11:20 a.m. Report left his work at Brgy. Central, and walked home, taking a
shows the following: short-cut.
-While passing at the vicinity of the Gonzales spouses Whether or not the client, under the evidence presented, has committed the
home at around 8:00 pm, he heard cries for help. felony of murder.
Curiosity prompted him to approach the place where the
shouts were from. HELD:

-15-20 m away from the scene, he hid himself behind a NO. Courts analysis of the evidence:
clump of banana trees, and saw all the accused ganging
upon the deceased near a threshing platform. He said he -Investigation conducted left much to be desired. Centeno gave
clearly recognized all the accused as the place was awash the date of commission as March 21, 1981. The sketch made was troubling,
in moonlight. as it did not effectively indicate the extent of the blood stains in the scenes
of crime. This would have added a lot of weight to any one of the versions
-After stabbing and hacking the victim, the accused lifted of the incident
his body and carried it to the house. Huntoria then left
home. Upon reaching his house, he related what he saw to -Sazon, who claimed that Gonzales surrendered to him, failed to
his wife and mother before going to sleep. state clearly the reason for the surrender. It may even be possible that
Augusto surrendered just so he could be safe from the victims kin. Sazon
-Eight months after the incident, bothered by his also admitted that Augusto never mentioned to him the participation of
conscience and the fact that his father was a tenant of the other persons in the killing.
deceased, he thought of helping the widow. Out of his
own volition, he travelled to the widows house, and -Rojas statement showed two possibilities for the killing. Faustas
related to her what he saw admission that she was the only killer is plausible. Furthermore, there were
only five fatal wounds, which will be discussed later.
Except Fausta who admitted killing the deceased as he was trying
to rape her, the rest denied participation in the crime. The appellant claimed -Huntorias testimony, of which the prosecutions argument solely
that he was asleep in his house which was one kilometer away from the rests, needs to be examined further. Huntorias claims in his testimony did
scene of the crime, and he knew of the crime only when his grandchildren not exactly match with those from his cross-examination. He first claimed
went to his house that night that he recognized the people involved. However, in the cross-examination,
he only saw flashes. This implies that he may not have recognized anyone
The trial court disregarded the version of the defense; it believed at all.
the prosecutions version.

On appeal to the Court of Appeals, the appellant contended that the


trial court erred in convicting him on the basis of the testimony of the lone On the criminal liability of the appellant:
witness, and in not appreciating his defense of alibi. The Court found no -There is nothing in the findings or the evidence that establishes
merit in the errors, and rejected defense of alibi. Worsening this is that the the criminal liability of the appellant as a principal for direct participation
appellate court found the sentence erroneous, and upgraded the penalty to under Art. 17, para. 1 of the Revised PenalCode.
that of murder reclusion temporal/death. The case is now brought upon
certification by the Court of Appeals, hence the appeal -Furthermore, there is nothing in the findings or evidence that
inculpates him by inducement, under paragraph 2 of the same article. Based
ISSUE: on the definition of felonies in Art. 3 of the Revised Penal Code, the
prosecutions evidence could not establish intent nor fault. Recall that the aging parents are usually sheltered and insulated from possible harm. It is
elements of felonies include: improbable for the accused to bring their aging father when they were
clearly in better shape than he was, and it was unlikely for the appellant to
-An act or omission offer his services as they were more or less enough to handle what could
-Act or omission must be punishable have been a perceived enemy.
-Act is performed or omission incurred by deceit or fault
-The lone witness could not properly establish any acts or Although alibi is a weak defense, in cases like this where the
omissions done by the appellant. He stated that he does not know who participation of the appellant is not clear, it may be considered. In light of
hacked or stabbed the victim, thus implying that he does not know what the the evidence on record, it may be sufficient for an acquittal.
appellant did. With this, the essential elements of felonies may not even be
present. Decision of the CA is reversed and set aside. Appellant acquitted
-Furthermore, the fact that there were five stab wounds and six CASE No. 4
accused would imply tha one of them may not have caused a grave wound
(especially given the statement of the physician). This may have been the U.S. vs. DELOS REYES
appellant, and given that there is no evidence that the appellant caused any G.R. No. 504 September 16, 1902
of the wounds, coupled with the prosecutions failure to prove the presence
of conspiracy (that is, how many people actually took part in the killing), it
weakens the arguments against the appellant. On the lone witness:- DOCTRINE:
There can be no conviction under the Penal Code where by reason of a
mistake of fact the intention to commit the crime does not exist. However,
Huntorias credibility as a witness is tarnished by two points:
the diligence with which the law requires the individual at all times to
1. He came out eight months after the killing. He claims that he govern his conduct varies with the nature of the situation in which he is to
feared for his life, but there was no proof that he was being perform. In a matter so important to the good order of society as that in
threatened, nor was the length of time reasonable given the question, where the consequences of a mistake are necessarily so serious,
circumstances. nothing less than the highest degree of diligence will satisfy the standard
2. He is not exactly a disinterested/neutral witness. He admitted prescribed by the law.
to being a tenant of the deceased, and stated that one of the
FACTS:
reasons why he testified was because the victim was his
Julian Gonzales and Accused Tomasa Delos Reyes were married in 1897.
landlord.-
After some time, the two separated, with Gonzales living elsewhere while
-Under our socioeconomic set-up, a tenant owes the source of his Delos Reyes stayed at their home where Gonzales mother also lived. On
livelihood from his landlord. As such, they would do everything to get the July 12,1900, Delos Reyes remarried to Ramon Martinez. Thus, Gonzales
landlords to their favor. Posing as a witness would have been a convenient filed for a case of bigamy against his wife.
way to do this, especially as he ceased to be employed as early as May As her defense, she testified that she honestly believed that her husband was
1981. dead when she married Martinez. That sometime after they separated, the
mother of Gonzales informed her that her son was dead and she believed
Finally, based on Philippine customs and traditions, it is unlikely this to be true.
for the appellant to be in the scene of the crime, as under our family culture, As witness, Gonzales testified that after he left their house, he would still
visit his relatives who were left living in their house almost every day until The defendant, Ah Chong, was employed as a cook at Officers quarters.
a few days before the trial of this case. On the night, the defendant, who had received for the night, was suddenly
awakened by some trying to force open the door of the room. He sat up in
ISSUE: bed and called out twice, Who is there? He heard no answer and was
Was acquittal of the accused for the crime of bigamy proper? YES convinced by the noise at the door that it was being pushed open by
RULING: someone bent upon forcing his way into the room. The defendant, fearing
Acquittal for bigamy was proper, however, accused should be convicted that the intruder was a robber or a thief, leaped to his feet and called out: If
under Art. 568 (Old Penal Code: reckless imprudence) you enter the room, I will kill you. He was struck just above the knee by
the edge of the chair and he thought that the blow had been inflicted by the
person who had forced the door open, whom he supposed to be a burglar.
Seizing a common kitchen knife which he kept under his pillow, the
U.S. vs. Ah Chong (15 Phil. 488) defendant struck out wildly at the intruder who, it afterwards turned out,
Doctrine/Topic: Art. 3; Mental element (Mens rea) - Deliberate intent was his roommate. The roommate eventually died.
(Dolo) - Mistake of fact ISSUE: Whether or not Ah Chong may be held criminally responsible for
Had the facts been as Ah Chong believed them to be, he would have been murder in the case at bar.
justified in killing the intrude. Par. 1, Art XI of the Revised Penal Code People v Estrada
(RPC) provides that in order for the act to be justified, the requisites must
be present. Doctrines and Decision:

Requisites (Honest Mistake of Fact): The basic principle in our criminal law is that a person is criminally liable
for a felony committed by him. Under the classical theory on which our
1) Unlawful agression on the part of the victim penal code is mainly based, the basis of criminal liability is human free will.
Man is essentially a moral creature with an absolutely free will to choose
2) Reasonable necessity of the means employed to prevent or repel the between good and evil. When he commits a felonious or criminal act (delito
unlawful aggression doloso), the act is presumed to have been done voluntarily,i.e., with
freedom, intelligence and intent. Man, therefore, should be adjudged or held
3) Lack of sufficient provocation on the part of the person defending
accountable for wrongful acts so long as free will appears unimpaired.
himself
In the absence of evidence to the contrary, the law presumes that every
If the victim was really a robber, forcing his way into the room of Ah person is of sound mind and that all acts are voluntary. The moral and legal
Chong, there would have been unlawful aggression, there would have been presumption under our law is that freedom and intelligence constitute the
a necessity on the part of Ah Chong to defend himself and/or his home and normal condition of a person. This presumption, however, may be
the knife would have been a reasonable means to prevent or repel such overthrown by other factors; and one of these is insanity which exempts the
aggression. actor from criminal liability.
The act done by Ah Chong was merely an act done due to Honest Mistake ART. 12. Circumstances which exempt from criminal liability.The
of Fact. The Court acquits Ah Chong. following are exempt from criminal liability:

FACTS:
1. An imbecile or an insane person, unless the latter has acted at the time he is informed at the arraignment of the nature and cause of the
during a lucid interval. accusation against him, the process is itself a felo de se, for he can neither
comprehend the full import of the charge nor can he give an intelligent plea
When the imbecile or an insane person has committed an act which the law thereto.
defines as a felony (delito), the court shall order his confinement in one of
the hospitals or asylums established for persons thus afflicted, which he The fact that accused-appellant was able to answer the questions asked by
shall not be permitted to leave without first obtaining the permission of the the trial court is not conclusive evidence that he was competent enough to
same court. stand trial and assist in his defense. Section 12, Rule 116 speaks of an
unsound mental condition that effectively renders [the accused] unable to
An insane person is exempt from criminal liability unless he has acted fully understand the charge against him and to plead intelligently thereto. It
during a lucid interval. If the court therefore finds the accused insane when is not clear whether accused-appellant was of such sound mind as to fully
the alleged crime was committed, he shall be acquitted but the court shall understand the charge against him. It is also not certain whether his plea
order his confinement in a hospital or asylum for treatment until he may be was made intelligently. The plea of not guilty was not made by accused-
released without danger. An acquittal of the accused does not result in his appellant but by the trial court because of his refusal to plead.
outright release, but rather in a verdict which is followed by commitment of
the accused to a mental institution. Section 12, Rule 116 of the 1985 Rules on Criminal Procedure speaks of a
mental examination. The human mind is an entity, and understanding it is
In the eyes of the law, insanity exists when there is a complete deprivation not purely an intellectual process but depends to a large degree upon
of intelligence in committing the act. Mere abnormality of the mental emotional and psychological appreciation.Thus, an intelligent determination
faculties will not exclude imputability. The accused must be so insane as to of an accuseds capacity for rational understanding ought to rest on a deeper
be incapable of entertaining a criminal intent. He must be deprived of and more comprehensive diagnosis of his mental condition than laymen can
reason and act without the least discernment because there is a complete make through observation of his overt behavior. Once a medical or
absence of the power to discern or a total deprivation of freedom of the will. psychiatric diagnosis is made, then can the legal question of incompetency
Sec. 12. Suspension of arraignment.The arraignment shall be suspended, if be determined by the trial court. By this time, the accuseds abilities may be
at the time thereof: measured against the specific demands a trial will make upon him.

(a) The accused appears to be suffering from an unsound mental Facts:


condition which effectively renders him unable to fully understand That on or about the 27th day of December 1994 in the City of Dagupan,
the charge against him and to plead intelligently thereto. In such Philippines, the accused, Roberto Estrada, being then armed with a butchers
case, the court shall order his mental examination and, if necessary, knife, with intent to kill one Rogelio Mararac with treachery and committed
his confinement for such purpose. in a holy place of worship, did then and there, wilfully, unlawfully and
(b) x x x. criminally, attack, assault and use personal violence upon the latter by
stabbing him, hitting him on vital parts of his body with the said weapon,
The arraignment of an accused shall be suspended if at the time thereof he thereby causing his death shortly thereafter due to Cardiorespiratory Arrest,
appears to be suffering from an unsound mental condition of such nature as Massive Intrathoracic Hemorrhage, Stab Wound as per Autopsy Report and
to render him unable to fully understand the charge against him and to plead Certificate of Death. At the arraignment, accused-appellants counsel, filed
intelligently thereto. Under these circumstances, the court must suspend the an Urgent Motion to Suspend Arraignment and to Commit Accused to
proceedings and order the mental examination of the accused, and if Psychiatric Ward at Baguio General Hospital. It was alleged that accused-
confinement be necessary for examination, order such confinement and appellant could not properly and intelligently enter a plea because he was
examination. If the accused is not in full possession of his mental faculties suffering from a mental defect; that before the commission of the crime, he
was confined at the psychiatric ward of the Baguio General Hospital in
Baguio City. He prayed for the suspension of his arraignment and the
issuance of an order confining him at the said hospital. The motion was FACTS:
opposed by the City Prosecutor. The trial court, motu proprio, propounded
Petitoner Jesusa Cruz, presently confined at the Correctional
several questions on accused-appellant. Finding that the questions were
understood and answered by him intelligently, the court denied the motion Institution for Women in Mandaluyong City serving the penalty of life
that same day. imprisonment imposed upon her as a consequence of her conviction which
became final and executory last March 1, 1993 for violation of Section 4,
Issue: Article II of R.A 6425, filed, through her counsel, the present petition for
habeas corpus. Petitioner alleges that, as of the date of filing of her herein
Whether or not insanity of a person can exempt him from criminal liability
petition, she has already served five and a half years of her life
and postpone the trial for a case against him.
sentence. She argues that the penalty of life imprisonment imposed by the
Whether the judge through intelligent questions can validly determine the trial court is excessive considering that the marijuana allegedly taken from
mental capacity of a person in determining whether to continue with the her was only 5.5 grams or less than 750 grams.
trial and with his liability in a criminal case.

ISSUE: After having served five and a half years of her life sentence, may
JESUSA CRUZ VS CORRECTIONAL INSTITUTION FOR WOMEN petitioner -- who was convicted of selling 5.5 grams of prohibited drugs,
IN MANDALUYONG namely, dried marijuana leaves -- be now entitled to the beneficent penalty
G.R. NO 125672 Septmeber 27, 1996 provisions of R.A. 7659 and be now released from imprisonment?

RA 7659, which took effect on December 13, 1993, partly


modified the penalties prescribed by R.A. 6425 (Dangerous Drugs Act of Case #7
1972), that is, inter alia, where the quantity of prohibited drugs involved is People of the Philippines v. Daniel Quijada
G.R. Nos. 115008-09. July 24, 1996
less than 750 grams, the penalty is reduced to a range of prision
Article 14 Aggravating Circumstances
correccional to reclusion perpetua and that where the marijuana is less than
250 grams, the penalty to be imposed shall be prision correccional. Ruling:
Moreover, applying the Indeterminate Sentence Law, the penalty imposable The unequivocal intent of the second paragraph of Section 1 of P.D. No.
is further reduced to any period within arresto mayor, as minimum term, to 1866 is to respect and preserve homicide or murder as a distinct offense
the medium period of prision correccional as the maximum term, there penalized under the Revised Penal Code and to increase the penalty for
being no aggravating or mitigating circumstances illegal possession of firearm where such a firearm is used in killing a
person. Its clear language yields no intention of the lawmaker to repeal or
modify, pro tanto, Articles 248 and 249 of the Revised Penal Code, in such
The petitioner should now be deemed to have served the maximum a way that if an unlicensed firearm is used in the commission of homicide
period imposable for the crime for which she was convicted, i.e., selling 5.5 or murder, either of these crimes, as the case may be, would only serve to
grams of dried marijuana leaves. Although her penalty of life imprisonment aggravate the offense of illegal possession of firearm and would not
had already become final, the beneficial effects of the amendment provided anymore be separately punished. Indeed, the words of the subject provision
under R.A. 7659 should be extended to petitioner. are palpably clear to exclude any suggestion that either of the crimes of
homicide and murder, as crimes mala in se under the Revised Penal Code, is Facts:
obliterated as such and reduced as a mere aggravating circumstance in On or about the 30th day of December, 1992, in the municipality of Dauis,
illegal possession of firearm whenever the unlicensed firearm is used in province of Bohol, Philippines, the accused, with intent to kill and without
killing a person. The only purpose of the provision is to increase the penalty any justifiable motive, with treachery and abuse of superior strength, the
prescribed in the first paragraph of Section 1 -- reclusion temporal in its accused being then armed with a .38 cal. revolver, while the victim was
maximum period to reclusion perpetua -- to death, seemingly because of the unarmed, suddenly attacked the victim without giving the latter the
accused's manifest arrogant defiance and contempt of the law in using an opportunity to defend himself, and with evident premeditation, the accused
unlicensed weapon to kill another, but never, at the same time, to absolve having harbored a grudge against the victim a week prior to the incident of
the accused from any criminal liability for the death of the victim. murder, did then and there willfully, unlawfully and feloniously attack,
Neither is the second paragraph of Section 1 meant to punish homicide or assault and shoot Diosdado Iroy y Nesnea with the use of the said firearm,
murder with death if either crime is committed with the use of an unlicensed hitting the latter on his head and causing serious injuries which resulted to
firearm, i.e., to consider such use merely as a qualifying circumstance and his death; to the damage and prejudice of the heirs of the deceased. That the
not as an offense. That could not have been the intention of the lawmaker accused carried and had in his possession, custody and control a firearm
because the term "penalty" in the subject provision is obviously meant to be (hand gun) with ammunition, without first obtaining the necessary permit or
the penalty for illegal possession of firearm and not the penalty for license to possess the said firearm from competent authorities which firearm
homicide or murder. We explicitly stated in Tac-an: was carried by the said accused outside of his residence and was used by
There is no law which renders the use of an unlicensed him in committing the crime of Murder with Diosdado Iroy y Nesnea as the
firearm as an aggravating circumstance in homicide or victim.
murder. Under an information charging homicide or
murder, the fact that the death weapon was an Accused-appellant Daniel Quijada now appeals from the decision of the
unlicensed firearm cannot be used to increase the Regional Trial Court (RTC) of Bohol convicting him of the two offenses
penalty for the second offense of homicide or murder separately charged in two informations, viz., murder under Article 248 of
to death. The essential point is that the unlicensed the Revised Penal Code and illegal possession of firearm in its aggravated
character or condition of the instrument used in form under P.D. No. 1866, and imposing upon him the penalty of reclusion
destroying human life or committing some other perpetua for the first crime and an indeterminate penalty ranging from
crime, is not included in the inventory of aggravating seventeen years, four months, and one day, as minimum, to twenty years
circumstances set out in Article 14 of the Revised and one day, as maximum, for the second crime.
Penal Code.
Issue:
In short, there is nothing in P.D. No. 1866 that manifests, even vaguely, a Whether or not the RTC is correct in convicting the accused of two separate
legislative intent to decriminalize homicide or murder if either crime is crimes.
committed with the use of an unlicensed firearm, or to convert the offense
of illegal possession of firearm as a qualifying circumstance if the firearm People v. Pinto Jr.
so illegally possessed is used in the commission of homicide or murder. To
charge the lawmaker with that intent is to impute an absurdity that would JUSTIFYING CIRCUMSTANCE - In the performance of a Public
defeat the clear intent to preserve the law on homicide and murder and Duty
impose a higher penalty for illegal possession of firearm if such firearm is AGGRAVATING CIRCUMSTANCE - Evident Premeditation,
used in the commission of homicide or murder. Unlawful Aggression, Night time, Taking Advantage of Public Position
MOTIVE - Needed to prove Circumstantial Evidence
ABERATTIO ICTUS - Mistake in the Identity
CONSPIRACY - Acting in concert captives he saw a man with a bolo in his hand running towards him. As the
man was menacingly near him, Pinto shot the man later identified as
================================= Rosalio Andes when he was at a distance of around three meters
Pt. I
================================= Inocencia's husband was about to offer Bello a cup of coffee when she
heard a successive burst of gunfire. Bello, who was the balcony facing the
Search warrant was issued for inspecting the house and premises of copra kiln ("agonan") with his back towards the pili tree, gradually fell to
Francisco Bello on the ground that the police had probable cause that Bello the floor with his hands above his head. Then there was another burst of
illegally possessed a garand rifle, a thompson submachinegun and two gunfire. From the kitchen, Inocencia rushed to the door from where she saw
automatic pistols a man holding a long firearm, whom she later identified as Pinto, near the
pili tree which was around eight meters from where Bello was, and another
1 of 3 teams were walking on the Mariawa road toward the premises, when man, also holding a gun, crouching near the stairs
one PC Romero heard rumbling of a jeep, heard gunshots and saw flashes
of light from the direction of Buenaflor, his team member running towards Inocencia was about to rush to Bello. Just then a man, whom Inocencia
the same. identified as Buenaflor, came up the house, pointed a gun at Inocencia and
her husband and told them to lay flat on the floor.
On the jeep which passed by the deployed policemen were Fr. Felix
Cappellan, Mrs. Zenaida Stilianopolous Tiongson, her six children and the When Francisco Andes went up the house, he told Inocencia that Rosalio
driver returning from mass, fearing that there might be "people with bad was dead. Inocencia went near the pili tree where Rosalio's body was, knelt
intentions" or hold-uppers, Fr. Capellan told the driver to go faster. Shots down and asked the man with a long firearm why he killed Rosalio. The
were fired at the jeep. man answered that Rosalio fought back. However, Inocencia did not notice
any weapon near Rosalio's body
After the shooting incident, the police pursued the mission searching the
premises. =================================
Pt. III
================================= =================================
Pt. II
================================= Buenaflor and Pinto were charged: without any justifiable cause or motive,
Although Bello and his parents, Mr. and Mrs. Anduiza, were not around, the with intent to kill,
police searched the area and found a Japanese Springfield rifle, ammunition
of a garand rifle, ammunition of a carbine, live ammunition for a .38 caliber with treachery and evident premeditation, by means of a Cal. 45 Thompson
pistol and 380 bullets for an automatic pistol. Sub-Machine Gun and a US Carbin Inland, Cal. 30 owned respectively by
Thereafter, the Chief of Police declared the search terminated and the entire said accused,
searching party left for headquarters
(1) shoot one Rosalio Andes,23y/o. inflicting upon him gunshot wounds
Team 3 was instrued by a "superior officer" "to remain and maintain peace causing upon said Rosalio Andes serious and mortal wounds which led to
and order in (the) vicinity including Mariawa". While he and Buenaflor his instantaneous death.
were patrolling the area, at around midnight, they "chanced upon a house"
wherein Bello and his group were staying. They captured four of Bello's
bodyguards and tied them to a pili tree with the torn shirt of one of the
(2) one for the murder of Richard Tiongson, 9y/o. sustained a gunshot The police had obtained a search warrant for illegal possession of firearms
wound at the back, Richard was operated at the hospital but he died against Bello even on Christmas day which was supposed to be a holiday,
suffering a hemorrage. no such effort was made in securing warrant of arrest for Bello's alleged
frustrated killing of Botin
(3) another for the frustrated murder of Maria Theresa Tiongson.
(2) On Killing Bello and Andres:
(4) Bello died because of "shock secondary to massive hemorrhage due to Evident premeditation has not been proven beyond reasonable doubt in this
multiple gunshot wounds. case but we find that the appellants indeed took advantage of their public
position in perpetrating the crime.
=================================
ISSUES (3) As regards the unlawful aggression of Rosalio Andes against Pinto, we
================================= find that if we are to believe Pinto, we have to stamp full credibility on his
statement alone. Even Buenaflor admitted that he did not see Rosalio Andes
(1) Is the defense of fulfillment of a duty under Article 11(5) of the RPC attack Pinto.
tenable?
(2) Is there evident premeditation, treachery, night time, public position (4) On the jeep coming towards them was owned by the Anduizas, the
present as Aggravating Circumstances? appellants acted obviously in the belief that Bello was its passenger and
(3) Is the defense of unlawful aggression tenable? posthaste they fired upon it even without any inquiry as to the identity of its
(4) Is the defense of mistake in the identity of those who were shot in the passengers
jeep tenable?
Buenaflor's motive for wanting to do away with Bello has been established.
================================= Such motive provided a circumstantial evidence leading to the inference
RULING that indeed he fired his gun
=================================
All these pieces of circumstantial evidence point to no other inference than
(1) Originally set out to perform a legal duty: the service of a search warrant that Pinto and Buenaflor fired their guns in defiance of their superior
on Bello. In the process, however, appellants abused their authority officer's order only "to find the whereabouts" of Bello
resulting in unauthorized and unlawful moves and consequences. Armed
with only a search warrant and the oral order to apprehend Bello, they went The fact that the victims were different from the ones the appellants
beyond the ambit of their mission and deprived Bello and two other persons intended to injure cannot save them from conviction. Aberratio ictus or
of their lives. mistake in the identity of the victim carries the same gravity as when the
accused zeroes in on his intended victim. The main reason behind this
The defense has to prove that these two requisites are present: (a) the conclusion is the fact that the accused had acted with such a disregard for
offender acted in the performance of a duty and (b) the injury or offense the life of the victim(s) without checking carefully the latter's identity as
committed be the necessary consequence of the due performance or lawful to place himself on the same legal plane as one who kills another willfully,
exercise of such duty. In the absence of the second requisite, the unlawfully and feloniously. Neither may the fact that the accused made a
justification becomes an incomplete one thereby converting it into a mistake in killing one man instead of another be considered a mitigating
mitigating circumstance under Articles 13 and 69 of the same Code circumstance
It is not even necessary to pinpoint who between Pinto and Buenaflor Tecson while he was lying in bed.
actually caused the death of Richard or the wounding of Maria Theresa in
the presence of proof beyond reasonable doubt that they acted in conspiracy The RTC charged the defendants of Homicide through reckless impudence
with each other. and senteced each an indeterminate penatly and a fine to pay the heirs of
Tecson.
The lower court is AFFIRMED with MODIFICATIONS that appellants
shall solidarily be liable for each of the three murders they committed and, Defendants contended that they incur no criminal liability because they
for the frustrated murder of Maria Theresa Tiongson
believed that the man was Balagtas and they acted in innocent mistake of
fact in the honest performance of their official duties.
People vs. Oanis
Art. 11, no. 5 of RPC Issue: WON they should be held responsible for the death caused to
Art. 69 of RPC Tecson?
Murder
Rule 109, Sec. 2, par. 2 Held: Yes, they are responsible for tge death of Tecson.

Facts: The SC are of the opinion that the crime committed is murder though
In the afternoon of Dec. 24, 1938, The Constabulary Provincial inspector specially mitigated. The contention of the defendants that they are not liable
received a telegram stating " escaped convict Anselmo Balagtas with because of their defense of "mistake of fact" is not applicable.
bailarina and Irene in Cabanatuan get him dead or alive". The Inspector then
called his men, Included is Alberto Galanta, and instructed to arrest In the case of Ah Chong, it was shown that there was an innocent mistake of
Balagtas, and if overpowered, to follow the instruction contained in the fact committed without any fault or carelessness because the accused,
telegram. The same instructions were given to Chief of Polic Oanis, by the having no time or opportunity to make a further inquiry, and being pressed
Inspector. by circumstances to act immediately, had no alternative but to take the facts
as they then appeared to him.
The inspector divided the party into 2 groups to locate Balagtas. Galanta
and Oanis were group together and proceeded to Irene's house. Upon In the instant case, appellants found no circumstances whatsorever which
reaching the house, Oanis approached brigida Mallare and asked Irenes would press them to immediate action. The person in the room being asleep,
location. appellants had ample time and opportunity to ascertain his identity without
hazard to themselves, and could even effect a bloodless arrest if any
Defendants then went to the room of Irene and upon seeing a man sleeping reasonable effort to that end had been made.
with his back towards the door where they were were, simultaneously or
successively fired at him with their revolvers. It turned out that the person The SC also held that the crime committed by appellants is not merely
shot and killed was not the notorious Balagtas, but an innocent Serapio criminal negligence, the killing being intentional and not accidental. Stating
Tecson, who was Irene's paramour. that "a deloberate intent to di an unlawful act is essentially inconsistent with
the idea of reckless imprudence.
On trial, the testimonies of the defendants were contrary to each other but
such was corroborated by Irene's testimony that the defendants fired at There is, however, an incomplete justifying circumstance, falling under Art.
11 no. 5 of the RPC. Only one requisite was present as the "offenders acted
in the performance of a duty or in the lawful exercise of a right". Such being persons or property, were it not for the inherent impossibility of its
the case, and applying Art 69 od the RPC, the penalty lower by one or two accomplishment or on account of the employment of inadequate or
degress than that prescribed by law shall be imposed. ineffectual means.
Petitioner contends that, Palangpangan's absence from her room on the
night he and his companions riddled it with bullets made the crime
inherently impossible.
The Revised Penal Code, inspired by the Positivist School, recognizes in
the offender his formidability to punish criminal tendencies in Art. 4(2)
Legal impossibility occurs where the intended acts, even if completed,
Intod vs. Court of Appeals would not amount to a crime
Legal impossibility would apply to those circumstances where
1. the motive, desire and expectation is to perform an act in violation of
FACTS: the law
February 4, 1979: Sulpicio Intod, Jorge Pangasian, Santos Tubio and 2. there is intention to perform the physical act
Avelino Daligdig went to Salvador Mandaya's house and asked him to go 3. there is a performance of the intended physical act
with them to the house of Bernardina Palangpangan. Thereafter, they had a 4. the consequence resulting from the intended act does not amount to a
meeting with Aniceto Dumalagan who told Mandaya that he wanted crime
Palangpangan to be killed because of a land dispute between them and that o Ex: The impossibility of killing a person already dead
Mandaya should accompany them. Otherwise, he would also be killed. Factual impossibility occurs when extraneous circumstances unknown to
February 4, 1979 10:00 pm: All of them armed arrived at Palangpangan's the actor or beyond his control prevent the consummation of the intended
house and fired at Palangpangan's bedroom but there was no one in the crime this case
room. o Ex: man who puts his hand in the coat pocket of another with the
RTC: convicted Intod of attempted murder based on the testimony of the intention to steal the latter's wallet and finds the pocket empty
witness United States: where the offense sought to be committed is factually
impossible or accomplishment - attempt to commit a crime; legally
ISSUE: W/N Intod is guilty attempted murder since it is an impossible impossible of accomplishment - cannot be held liable for any crime
crime under Art. 4 (2)

HELD: YES. petition is hereby GRANTED, the decision of respondent Case # 12


Court of Appeals holding Petitioner guilty of Attempted Murder is hereby
MODIFIED. sentences him to suffer the penalty of six (6) months of arresto ESCAMILLA V. PEOPLE
mayor, together with the accessory penalties provided by the law, and to pay
the costs G.R. No. 188551 February 27, 2013

HOMICIDE
Art. 4(2). CRIMINAL RESPONSIBILITY. Criminal Responsibility
shall be incurred: DOCTRINE: The intent to kill, as an essential element of homicide at
xxx xxx xxx whatever stage, may be before or simultaneous with the infliction of
2. By any person performing an act which would be an offense against injuries. The evidence to prove intent to kill may consist of, inter alia, the
means used; the nature, location and number of wounds sustained by the 13. People vs Campuhan [G.R. No. 129433. March 30, 2000]
victim; and the conduct of the malefactors before, at the time of, or
immediately after the killing of the victim.l1 Ruling: NO. MODIFIED. guilty of ATTEMPTED RAPE and sentenced to
an indeterminate prison term of eight (8) years four (4) months and ten (10)
days of prision mayor medium as minimum, to fourteen (14) years ten (10)
Petitioners intent to kill was simultaneous with the infliction of injuries.
months and twenty (20) days of reclusion temporal medium as maximum.
Using a gun, he shot the victim in the chest, petitioner continued to shoot at
Costs de oficio.
him three more times, albeit unsuccessfully. While running, the victim saw
his nephew in front of the house and asked for help. The victim was
People v. De la Pea: labia majora must be entered for rape to be
immediately brought to the hospital. The doctor said that the victim would
consummated
have died if the latter were not brought immediately to the hospital. All
Primo's kneeling position rendered an unbridled observation impossible
these facts belie the absence of petitioners intent to kill the victim.
Crysthel made a categorical statement denying penetration but her
vocabulary is yet as underdeveloped
FACTS: Corazon narrated that Primo had to hold his penis with his right hand,
1. Around 2am of 01 August 1999, a brawl ensued at the comer of Estrada thus showing that he had yet to attain an erection to be able to penetrate his
and Arellano Streets, Manila. Mendol was about to ride his tricycle at this victim
intersection while facing Arellano Street. the possibility of Primo's penis having breached Crysthel's vagina is
belied by the child's own assertion that she resisted Primo's advances by
2. Petitioner, who was standing in front of his store, 30 meters away from putting her legs close together and that she did not feel any intense pain but
Mendol, shot the later four times, hitting him once in the upper right portion just felt "not happy" about what Primo did to her. Thus, she only shouted
of his chest. "Ayo'ko, ayo'ko!" not "Aray ko, aray ko!
no medical basis to hold that there was sexual contact between the
3. The victim was brought to the Ospital ng Makati for treatment and
accused and the victim.
survived because of timely medical attention.
FACTS:
4. Assistant City Prosecutor of Manila Fled an Information charging
April 25, 1996 4 pm: Ma. Corazon P. Pamintuan, mother of 4-year old
petitioner with frustrated homicide.
Crysthel Pamintuan, went to the ground floor of their house to prepare Milo
5. During trial, the prosecution presented the testimonies of Mendol, Joseph chocolate drinks for her 2 children. There she met Primo Campuhan, helper
Velasco (Velasco) and Iluminado Garcelazo(Garcelazo), who all positively of Conrado Plata Jr., brother of Corazon, who was then busy filling small
identified the petitioner as the shooter of Mendol. plastic bags with water to be frozen into ice in the freezer located at the
second floor.
RTC: found petitioner guilty of frustrated homicide. Then she heard Crysthel cry, "Ayo'ko, ayo'ko!" so she went upstairs and
saw Primo Campuhan inside her children's room kneeling before Crysthel
CA: affirmed the conviction
whose pajamas or "jogging pants" and panty were already removed, while
Issue: Whether or not the CA is correct in affirming the conviction of his short pants were down to his knees and his hands holding his penis with
frustrated homicide? his right hand
Horrified, she cursed "P - t - ng ina mo, anak ko iyan!" and boxed him
several times. He evaded her blows and pulled up his pants. He pushed
Corazon aside who she tried to block his path. Corazon then ran out and bag where all the proceeds of his sale for the day are placed. While walking
shouted for help thus prompting Vicente, her brother, a cousin and an uncle with his 2 companions, Libantino and Galvez, 4 men accosted them. Two of
who were living within their compound, to chase the Campuhan who was the men held Ching and Galvez while Labantino continued to walk in front
apprehended. They called the barangay officials who detained. of them, initially unaware of the commotion. A .45 caliber pistol as pointed
Physical examination yielded negative results as Crysthel s hymen was at Ching while he was held, while another man snatched the paper bag from
intact him. When Ching tried to call Labantinos name for help, the gun was shot
Campuhan: Crysthel was in a playing mood and wanted to ride on his resulting to a gun wound on Ching. Labantino fled, while Galvez finally
back when she suddenly pulled him down causing both of them to fall down freed himself from the men holding him. The men eventually fled too. After
on the floor. falling from the gun shot, Ching was able to walk to home. His common-
RTC: guilty of statutory rape, sentenced him to the extreme penalty of law wife brought him to the hospital. He died a day after.
death
Thus, subject to automatic review Galvez and Labantino both identified Villacorte as one of the men who
accosted them. Upon interrogation, Villacorte admitted to the crime and
ISSUE: W/N it was a consummated statutory rape identified his companions as "Roque", "Sante" and "Fred".

Consequently, an information for robbery with homicide was filed against


Villacorte, Roque Guerrero (Roque), Alfredo Handig (Fred) and an alias
PEOPLE OF THE PHILIPPINES, plaintiff-appellee, vs. VIOLETO
Sante whose identity was not yet ascertained. The information was
VILLACORTE, alias BONGING, et al., defendants. CRISANTO
eventually amended to implicate Crisanto Inoferio. The trial court acquitted
INOFERIO Y ALINDAO alias SANTE, and MARCIANO YUSAY alias
Handig and found Villacorte and Inoferio guilty. Guerrero was discharged
MANCING (appeal withdrawn res. of 7/10/67), defendants-appellants.
and used as a state witness.
G.R. No. L-21860 February 28, 1974

Only Inoferio pursued this appeal. Inoferio testified that he first met
RULING: Although inherently weak and easily fabricated, the evidence
Villacorte and Handig while in Police custody for the investigation. He
presented by an accused in support of that defense must be scrutinized with
claims that he knew Guerrero as they played cara y cruz together.
the same care that evidence supporting other defenses deserves. When an
accused puts up the defense of alibi, the court should not at once have a
mental prejudice against him. For, taken in the light of all the evidence on ISSUE: Whether or not Inoferios weak alibi could warrant a conviction.
record, it may be sufficient to acquit him, as in the case of appellant
Inoferio. He should be acquitted upon the ground that although his defense, THE PEOPLE OF THE PHILIPPINES vs. AMADEO PERALTA, ET
in the nature of an alibi, is inherently a weak defense, it should be AL.
considered sufficient as in this case, to tilt the scale of justice in favor of the
Doctrine: A conspiracy exists when two or more persons come to an
accused because the evidence for the prosecution is itself weak and
agreement concerning the commission of a felony and decide to commit it.
unconvincing and, therefore, by and large, insufficient to prove the guilt of
Generally, conspiracy is not a crime except when the law specifically
the accused beyond reasonable doubt.
provides a penalty therefore as in treason, rebellion and sedition.

FACTS: After closing his shop for the day, Chinese merchant Benito Ching Proof of conspiracy. Conspiracy presupposes the existence of a
left his store in the public market of Caloocan. He brought with him a paper preconceived plan or agreement; however, to establish conspiracy, "it is not
essential that there be proof as to previous agreement to commit a crime, it common design to commit a felony; it is not participation in all the details
being sufficient that the malefactors committed shall have acted in concert of the execution of the crime. All those who in one way or another help and
pursuant to the same objective." Hence, conspiracy is proved if there is cooperate in the consummation of a felony previously planned are co-
convincing evidence to sustain a finding that the malefactors committed an principals. Hence, all of the six accused are guilty of the slaughter of
offense in furtherance of a common objective pursued in concert. Carriego, Barbosa and Santos Cruz each is guilty of three separate and
distinct crimes of murder.
Liability of conspirators. A time-honored rule in the corpus of our
jurisprudence is that once conspiracy is proved, all of the conspirators who Facts: On February 16, 1958, while the inmates of the penitentiary were
acted in furtherance of the common design are liable as co-principals. This preparing to attend Sunday mass, that a fight between two rival members of
rule of collective criminal liability emanates from the ensnaring nature of the "Sigue-Sigue" and "OXO" gangs occurred in the plaza where the
conspiracy. The concerted action of the conspirators in consummating their prisoners were assembled, causing a big commotion. The fight was,
common purpose is a patent display of their evil partnership, and for the however, quelled, and those involved were led away for investigation, while
consequences of such criminal enterprise they must be held solidarity liable. the rest of the prisoners were ordered to return to their respective quarters.
Moments later, another riot erupted in Bldg. 4, as the inmates of brigade 4-
Imposition of multiple penalties where conspirators commit A, mostly "OXO" members and sympathizers, destroyed the lock of their
more than one offense. Since in conspiracy, the act of one is the act of all, door and then rampaged from one brigade to another. The invading OXO
then, perforce, each of the conspirators is liable for all of the crimes clubbed and stabbed to death Jose Carriego, an inmate of 4-B. Afterwards,
committed in furtherance of the conspiracy. Consequently, if the they forcibly opened the door of 4-C and killed two more inmates, namely,
conspirators commit three separate and distinct crimes of murder in Eugenio Barbosa and Santos Cruz. The three victims sustained injuries
effecting their common design and purpose, each of them is guilty of three which swiftly resulted in their death before they could be brought to the
murders and shall suffer the corresponding penalty for each offense. hospital.
Legality and practicality of imposing multiple death penalties
upon conspirators. An accused who was charged with three distinct crimes
of murder in a single information was sentenced to two death penalties for Issue: Whether or not the commission of the murders was attended
two murders, and another accused to thirteen (13) separate death penalties with conspiracy?
for the 13 killings he perpetrated. Therefore there appears to be no legal
reason why conspirators may not be sentenced to multiple death penalties
corresponding to the nature and number of crimes they commit in PEOPLE OF THE PHILIPPINES, plaintiff-appellee, vs. EDWIN DE
furtherance of a conspiracy VERA yGARCIA, RODERICK GARCIA y GALAMGAM,
KENNETH FLORENDO and ELMER CASTRO, accused, EDWIN DE
Ruling. The evidence on record proves beyond peradventure that the
VERA yGARCIA, appellant.
accused acted in concert from the moment they bolted their common
brigade, up until the time they killed their last victim, Santos Cruz. While it Doctrines:
is true that Parumog, Larita and Luna did not participate in the actual killing
of Carriego, nonetheless, as co-conspirators they are equally guilty and Conspiracy - It is axiomatic that the prosecution must establish conspiracy
collectively liable for in conspiracy the act of one is the act of all. It is not beyond reasonable doubt. Mere presence does not amount to conspiracy
indispensable that a co-conspirator should take a direct part in every act and criminal conspiracy must be founded on facts, not on mere surmises or
should know the part which the others have to perform. Conspiracy is the conjectures.
Accomplices; Penalties;
Requisites of Conspiracy: The penalty of an accomplice is one degree lower than that of a principal.
To prove conspiracy, the prosecution must establish the following three
requisites: Murder; Aggravating Circumstances; Treachery; Abuse of Superior
1 that two or more persons came to an agreement; Strength; Treachery absorbs abuse of superior strength
2 that the agreement concerned the commission of a crime; and Treachery absorbs abuse of superior strength. The trial court found that the
3 that the execution of the felony [was] decided upon. killing was attended by treachery, evident premeditation and abuse of
superior strength. Hence, there is only one generic aggravating
Accomplice, Defined; Elements. circumstance, not two.
The Court has held that an accomplice is one who knows the criminal
design of the principal and cooperates knowingly or intentionally therewith Facts:
by an act which, even if not rendered, the crime would be committed just The accused with evident premeditation, treachery and use of superior
the same. strength, attack, assault and employ personal violence upon the person of
one FREDERICK CAPULONG y DIZON, hitting him between his eyes
To hold a person liable as an accomplice, two elements must be present: and striking him with the use of a baseball bat in the mouth, thereby
1 the community of criminal design; that is, knowing the criminal inflicting upon him serious and mortal wounds. The trial court ruled that
design of the principal by direct participation, he concurs with the there was conspiracy and sentenced all of the accused to suffer reclusion
latter in his purpose; and perpetua mainly on the testimony of an eyewitness. Specifically, it based its
2 the performance of previous or simultaneous acts that are not conclusions on the following facts: appellant was seen with the other
indispensable to the commission of the crime. accused inside the victims car; the victim was clearly struck with a blunt
object while inside the car, and it was unlikely for Florendo to have done it
Conspirator and Accomplice, Distinguished. all by himself; moreover, it was impossible for De Vera and Garcia to have
1 Once conspiracy is proven, the liability is collective and not been unaware of Florendos dark design on Roderick.
individual. The act of one of them is deemed the act of all. In the
case of an accomplice, the liability is one degree lower than that of Such witness, Cacao, testified that he saw Appellant De Vera in the car,
a principal. where an altercation later occurred. Thereafter, he saw Florendo drag out of
2 Conspirators and accomplices have one thing in common: they the vehicle an apparently disabled Capulong and shoot the victim in the
know and agree with the criminal design. Conspirators, however, head moments later. But, Cacaos testimony contains nothing that could
know the criminal intention because they themselves have decided inculpate appellant De Vera aside from the fact that he was inside the car, no
upon such course of action. Accomplices come to know about it other act was imputed to him.
after the principals have reached the decision, and only then do
they agree to cooperate in its execution. Issues:
3 Conspirators decide that a crime should be committed; 1 WON the trial court erred in deciding that there is conspiracy?
accomplices merely concur in it. Accomplices do not decide 2 WON the trial court erred in aside from treachery, there are other
whether the crime should be committed; they merely assent to the two aggravating circumstances (evident premeditation and abuse
plan and cooperate in its accomplishment. of superior strength).
4 Conspirators are the authors of a crime; accomplices are merely
their instruments who perform acts not essential to the perpetration Ruling:
of the offense. 1 YES. The prosecution must establish conspiracy beyond
reasonable doubt. Here, the bare testimony of Cacao fails to do so.
Cacaos testimony contains nothing that could inculpate appellant. A treacherous attack is one in which the victim was not afforded any
Aside from the fact that he was inside the car, no other act was opportunity to defend himself or resist the attack. The existence of
imputed to him. Mere presence does not amount to conspiracy. treachery is not solely determined by the type of weapon used. If it
Criminal conspiracy must be founded on facts, not on mere appears that the weapon was deliberately chosen to insure the execution
surmises or conjectures of the crime, and to render the victim defenseless, then treachery may
2 YES. Treachery absorbs abuse of superior strength. Hence, there is be properly appreciated against the accused.
only one generic aggravating circumstance, not two
PEOPLE OF THE PHILIPPINES, Appellee, v. REGIE LABAIGA,
Appellant.
FACTS
G.R. No. 202867, July 15, 2013

According to the Prosecution, Regie Labiaga shot Gregorio Conde outside


ARTICLE PRINCIPLE OF LAW the latters house. Conde cried for help which prompted his daughters Judy
and Glenelyn to rush towards him. Regie Labiaga shot Judy and told the
Article 6 Consummated, frustrated, and attempted felonies.
other accused Balatong Barcenas and Cristy Demapanag that she is already
dead., and they left the vicinity. Judy and Gregorio were rushed to the
In Serrano v. People, we distinguished a frustrated felony from an
hospital wherein Gregorio was treated and fully recovered Judy was
attempted felony in this manner: declared dead on arrival due to cardiopulmonary arrest secondary to
1 In a frustrated felony, the offender has performed all the acts
Cardiac Tamponade due to gunshot wound.
of execution which should produce the felony as a
consequence; whereas in an attempted felony, the offender According to the defense, Regie Labiaga admitted that he was present
merely commences the commission of a felony directly by during the shooting incident and claimed that he acted in self-defense when
overt acts and does not perform all the acts of execution. Gregorio challenged him to a fight while armed with a shotgun. He alleged
2 In a frustrated felony, the reason for the non-accomplishment that when he tried to wrest the gun from Gregorio, the shotgun fired without
of the crime is some cause independent of the will of the him knowing whether somebody was hit. Demapanag alleged that she was
perpetrator; on the other hand, in an attempted felony, the 14 km away from the crime scene which was corroborated by her brother.
reason for the non-fulfillment of the crime is a cause or
accident other than the offenders own spontaneous The accused were charged with Murder with the Use of Unlicensed
desistance. Firearm. The information stated that the accused, conspiring,
confederating and helping one another, armed with unlicensed firearm, with
In frustrated murder, there must be evidence showing that the wound deliberate intent and decided purpose to kill, by means of treachery and
would have been fatal were it not for timely medical intervention. If the with evident premeditation, did then and there willfully, unlawfully and
evidence fails to convince the court that the wound sustained would feloniously attack, assault and shoot Gregorio Conde with said unlicensed
have caused the victims death without timely medical attention, the firearm, x x x thereby performing all the acts of execution which would
accused should be convicted of attempted murder and not frustrated produce the crime of Murder as a consequence, but nevertheless did not
murder. produce it by reason of causes independent of the will of the accused; that
is by the timely and able medical assistance rendered to said Gregorio
Conde which prevented his death.
The RTC acquitted Demapanag due to insufficiency of evidence while Defense of one's person or rights is treated as a justifying circumstance
Regie Labiaga was convicted of murder and frustrated murder. CA-Cebu under Art. 11, par. 1 of the Revised Penal Code, but in order for it to be
affirmed the conviction. appreciated, the following requisites must occur:

First. Unlawful aggression;


ISSUES:
Second. Reasonable necessity of the means employed to prevent or repel it;
Whether Regie Labiaga is guilty of frustrated or attempted murder.
Third. Lack of sufficient provocation on the part of the person defending
himself (Art. 11, par. 1, Revised Penal Code, as amended).
HELD:

In the instant case, it does not appear that the wound sustained by Gregorio
Conde was mortal. This was admitted by Dr. Edwin Figura, who examined
Gregorio after the shooting incident. In this case, there was an aggression, not on the person of appellant, but on
his property rights when the victims angrily ordered the continuance of the
Since Gregorios gunshot wound was not mortal, the SC held that appellant fencing. There was also no provocation at all since the appellant was asleep
should be convicted of attempted murder and not frustrated murder. Under before the commission of the crime. However, the second element is not
Article 51 of the Revised Penal Code, the corresponding penalty for present as the killing was disproportionate to the attack.
attempted murder shall be two degrees lower than that prescribed for
consummated murder under Article 248, that is, prision correccional in its
maximum period to prision mayor in its medium period.
Facts:
Case No. 18
On August 22, 1968, Narvaez shot Fleischer and Rubia as the two were
constructing a fence that would prevent Narvaez from getting into his house
People vs. Narvaez and rice mill. The defendant was taking a nap when he heard sounds of
construction and found fence being made. He tried to stop the group from
121 SCRA 389 destroying his house and asked if they could talk things over but Fleischer
responded with "No, gadamit, proceed, go ahead." Defendant lost his
G.R. Nos. L-33466-67 "equilibrium," and shot Fleisher and Rubia with his shotgun. Defendant
claims he killed in defense of his person and property but the CFI ruled that
Narvaez was guilty and sentenced him to reclusion perpetua, to indemnify
April 20, 1983 the heirs, and to pay for moral damages.

Issue:
Whether or not the act of killing of the accused is in defense of his person
Intro: and of his rights, and therefore he should be exempt from criminal liability.
Flores vs People G.R. No. 181354, February 27, 2013 But Flores continued shooting Jesus. Considering the number of gunshot
wounds sustained by the victim, the Court finds it difficult to believe that
Article #/Doctrine: To successfully claim self-defense, the accused must Flores acted to defend himself to preserve his own life. "It has been held in
satisfactorily prove the concurrence of the elements of self-defense. Under this regard that the location and presence of several wounds on the body of
Article 11 of the Revised Penal Code, any person who acts in defense of the victim provide physical evidence that eloquently refutes allegations of
his person or rights does not incur any criminal liability provided that self-defense."37
the following circumstances concur: (1) unlawful aggression; (2) Facts: On August 15, 1989, on the eve of the barangay fiesta in San Roque,
reasonable necessity of the means employed to prevent or repel it; and Alaminos, Laguna, certain visitors, Ronnie de Mesa, Noli de Mesa, Marvin
(3) lack of sufficient provocation on the part of the person defending Avenido, and Duran, were drinking at the terrace of the house of Jesus.
himself. They started drinking at 8:30 oclock in the evening. Jesus, however, joined
his visitors only at around 11:00 oclock after he and his wife arrived from
The most important among all the elements is unlawful aggression. "There Sta. Rosa, Laguna, where they tried to settle a problem regarding a
can be no self-defense, whether complete or incomplete, unless the victim vehicular accident involving one of their children. The drinking at the
had committed unlawful aggression against the person who resorted to self- terrace was ongoing when Flores arrived with an M-16 armalite rifle.4
defense."30 "Unlawful aggression is defined as an actual physical assault, or
at least a threat to inflict real imminent injury, upon a person. In case of Duran testified that Jesus stood up from his seat and met Flores who was
threat, it must be offensive and strong, positively showing the wrongful heading towards the terrace. After glancing at the two, who began talking to
intent to cause injury. It presupposes actual, sudden, unexpected or each other near the terrace, Duran focused his attention back to the table.
imminent dangernot merely threatening and intimidating action. It is Suddenly, he heard several gunshots prompting him to duck under the table.
present only when the one attacked faces real and immediate threat to ones Right after the shooting, he looked around and saw the bloodied body of
life."31"Aggression, if not continuous, does not constitute aggression Jesus lying on the ground. By then, Flores was no longer in sight.5
warranting self-defense."32
Duran immediately helped board Jesus in an owner-type jeep to be brought
In this case, Flores failed to discharge his burden. Indeed, the nature and to a hospital. Thereafter, Duran, Ronnie de Mesa and Noli de Mesa went
number of the gunshot wounds inflicted upon Jesus further negate the claim home. Jesus was brought to the hospital by his wife and children. Duran did
of self-defense by the accused. Records show that Jesus suffered four (4) not, at any time during the occasion, notice the victim carrying a gun with
gunshot wounds in the different parts of his body, specifically: on the him.6
medial portion of the left shoulder, between the clavicle and the first rib; on
the left hypogastric region through the upper right quadrant of the abdomen; Gerry narrated that he was going in and out of their house before the
on the tip of the left buttocks to the tip of the sacral bone or hip bone; and shooting incident took place, anxiously waiting for the arrival of his parents
on the right flank towards the umbilicus. from Sta. Rosa, Laguna. His parents were then attending to his problem
regarding a vehicular accident. When they arrived, Gerry had a short
According to Dr. Ruben Escueta, who performed the autopsy on the victim, conversation with his father, who later joined their visitors at the terrace. 7
the latter died of massive intra-abdominal hemorrhage due to laceration of Gerry was outside their house when he saw Flores across the street in the
the liver.36 If there was any truth to Flores claim that he merely acted in company of some members of the CAFGU. He was on his way back to the
self-defense, his first shot on Jesus shoulder, which already caused the house when he saw Flores and his father talking to each other from a
latter to fall on the ground, would have been sufficient to repel the attack distance of about six (6) meters. Suddenly, Flores shot his father, hitting him
allegedly initiated by the latter. on the right shoulder. Flores continued shooting even as Jesus was already
lying flat on the ground. Gerry testified that he felt hurt to have lost his
father.8
unlawful aggression by the victim; (2) that the means employed to prevent
Issue: WHETHER THE SANDIGANBAYAN, FIRST DIVISION, or repel such aggression were reasonable; and (3) that there was lack of
GRAVELY ERRED IN NOT GIVING DUE CREDIT TO PETITIONERS sufficient provocation on the part of the person defending himself.
CLAIM OF SELF-DEFENSE.
If the appellants stabbed Carlos Catorse and Marcelo Lo to defend
themselves it certainly defies the reason why they had to inflict 16 stab
wounds and 6 respectively. The location, number and gravity of the wounds
inflicted on the victims belie the appellants contention that they acted in
[G.R. No. 109614-15. March 29, 1996] self-defense. The nature and extent of the wounds inflicted on a victim
negate an accused claim of self-defense.
PEOPLE OF THE PHILIPPINES, plaintiff-appellee, vs. ADRONICO
GREGORIO and RICARDO GREGORIO,defendants-
appellants.
FACTS

Around 8:00 PM of May 7, 1986, Carlos Catorse together with his


fifteen year old son Romeo Catorse arrived at the house of appellant
ARTICLE
Adronico Gregorio at Sitio Bug-as, Barangay Sta. Cruz, Murcia, Negros
Occidental, to attend the wake of the latters grandson.
ARTICLE 11. JUSTIFYING CIRCUMSTANCES The following do
Persons attending the wake were requested by appellant Adronico to deposit
not incur any criminal liability:
with him any weapon in their possession for safekeeping so as to avoid
1. Anyone who acts in defense of is person or rights, provided the following trouble. Complying therewith, Carlos Catorse handed over his samurai,
circumstances concur: John Villarosa and Remolito Calalas, their respective knives, to Adronico .
First. Unlawful aggression; Around 1:00 oclock in the morning of May 8, 1986, while the game of
Second. Reasonable necessity of the means employed to prevent or pusoy was still in progress, appellant Ricardo, in a very loud voice,
repel it; reprimanded Tunggak (son of Adronico) from peeping at the cards of other
Third. Lack of sufficient provocation on the part of the person players. In response, Tunggak stood up and also in a very loud voice
defending himself. ordered the game stopped).

Overhearing the incident, Adronico ordered his son downstairs and right
According to the court the guiding jurisprudential principle has there and then scolded and boxed him several times. While Adronico was
always been that when an accused invokes the justifying circumstance of severely beating Tunggak, Carlos Catorse approached and begged Adronico
self-defense, the burden of proof is shifted to him to prove the elements of from further hurting his son so as not to put him to shame before the
that claim; otherwise, having admitted the killing, conviction is inescapable. crowd. Carlos was in this act of pacifying the matter between the father and
And that he must rely on the strength of his own evidence and not on the son when suddenly appellant Ricardo stealthily stabbed Carlos from behind
weakness of the prosecution. Having admitted the killing, appellant has to with a samurai (the same samurai deposited by Carlos to Adronico) and
justify his taking of a life by the exacting standards of the law. For self- thereafter hacked and stabbed him several times more in different parts of
defense to prosper, the following requisites must concur: (1) there must be
his body: Right after Carlos fell to the ground, Adronico, for his part, safety. In the instant case, there was no imminent and real danger to the life
repeatedly hacked the victim with a bolo. or limb of the petitioner when he shot the deceased, since the latter had
already been disarmed. As former Chief Justice Aquino states in his book on
Romeo Catorse, son of Carlos Catorse, ran out of the house. Jovito Nicavera Criminal Law:
also tried to get out of the house but Adronico hacked him instead with a
bolo hitting his left shoulder. Marcelo Lo tried to help his uncle Jovito but
Ricardo, with the same samurai used against Carlos hacked him on his
forearm. Adronico immediately followed and using a bolo hacked Marcelo
In order to justify self-defense, it is essential that the attack upon
on the nape. Although wounded, Marcelo was able to run out of the house
defendant be simultaneous with the killing, or preceded the latter
but Adronico ran after and overtook him. Adronico then hacked him again.
without an appreciable interval of time. (Ferrer, 1 Phil. 56),
Ricardo died during the trial. Adronico on the other hand interposed self-
defense to exculpate himself from criminal liability.

ISSUE xxx xxx xxx


Whether or not the appellant can validly invoke self-defense.

The harm caused by one person to another who offended or caused


P/CPL. FAUSTO ANDAL v. SANDIGANBAYAN AND PEOPLE OF him injury, sometime after he suffered such offense or such injury,
THE PHILIPPINES does not constitute an act of self-defense, but an act of revenge.
(Banzuela 31 Phil. 564)
G.R. No. L-60159, 06 November 1989

In imposing on the appellant the penalty of just one (1) year of prision
correccional, the respondent Court held (which the Supreme Court
affirmed):
Introduction (Provisions involved, principles, jurisprudence):

xxx xxx xxx


The petitioner failed to prove the defense he had raised. The primordial
requisite of self-defense is unlawful aggression. And for unlawful
aggression to be present, there must be a real danger to life or personal
Article 69 of the Revised Penal Code vests discretion to the court charge and Quinio told Macaraig that the petitioner did not utter defamatory
in lowering the penalty either by one or two degrees whenever words against him and asked him to forget the incident. But still Macaraig
incomplete justifying circumstance exists in a given case like the challenged the petitioner and fired his gun at the petitioner, hitting the latter
case at bar. The laudable patience of accused in not retaliating in the middle aspect, lower right knee. The petitioner was able to wrest the
despite repeated insults by a subordinate, his length of service in gun from Macaraig when they grappled for the possession of the gun and
the government (since 1957), and most important, his obsession to two (2) successive shots were fired at Macaraig by the petitioner which
inculcate discipline in his men, to OUR mind, entitle accused to a caused Macaraigs death.
two-degree reduction of the penalty prescribed by law. Our attitude
is a signal to the men in uniform that while WE condemn felonious
violence WE support efforts to maintain discipline in the service.

Brief issue:

Brief facts:
Was there an appreciable time lapse between the first aggression, i.e. when
deceased shot accused on his knee and the time accused resorted to force by
way of firing the two shots at the deceased? The facts unfolded indicate that
At about 0700pm on 25 September 1980, Petitioner Fausto Andal, then a there was. This is what happened after accused had grabbed the gun: (1) He
corporal in the Batangas Integrated National Police, while on patrol aboard asked deceased, "Why did you fire at me?" (2) He even turned his head
a tricycle driven by Police Pfc. Casiano Quinio, went to the police station at towards his son and instructed him just to stay in the jeep. (3) His son,
the pier located at Sta. Clara, Batangas City to check on one of his men, Pfc. Domingo Andal, challenged deceased to a fight "Sportsman like." (4)
Maximo Macaraig, who was stationed there, because the said Macaraig had Deceased moved backward 2 meters away from accused. (5) Pfc. Quinio
failed to report to police headquarters for briefing but Macaraig replied that even thought the trouble was over as he started to get his tricycle
he did not have to report to police headquarters since he already had his
orders and told the petitioner: "You report, supsup, ka. Case no. 25

When petitioner Fausto Andal and Quinio went back to the poblacion of PEOPLE OF THE PHILIPPINES vs. GERARDO SAZON, alias
Batangas City and parked their vehicle in front of the Philbanking Building "INSIK,"
at P. Burgos Street, Batangas City, Macaraig arrived and went straight to the
petitioner as he was furious and demanded why the petitioner had G.R. No. 89684 | September 18, 1990
embarrassed him in front of so many people. The petitioner denied the
REGALADO, J. Conspiracy; The fact that appellant did not inflict the mortal wound upon
the deceased is of no moment, since the existence of conspiracy was
satisfactorily shown by the evidence. The coordinated acts of appellant and
Altejos of immediately following the victim and jointly confronting him
DOCTRINE / RULING: thereafter reveal a concordance and unity of thought which resulted in the
Self-Defense; Well-entrenched is the rule that where the accused invokes encounter. The circumstances that after the accused shot the victim in the
self-defense, it is incumbent upon him to prove by clear and convincing forearm and, while he and the victim were grappling for appellants gun,
evidence that he indeed acted in defense of himself. He must rely on the Altejos stabbed the victim to death, indicate closeness and coordination of
strength of his own evidence and not on the weakness of the prosecution. their action geared towards a common purpose, that is, to kill the victim.
For, even if the prosecution evidence is weak, it could not be disbelieved Proof of a previous agreement to commit the crime is not absolutely
after the accused himself had admitted the killing. essential to establish a conspiracy. It is sufficient that the accused be shown
to have acted in concert pursuant to the same objective, as such
It is a statutory and doctrinal requirement that for the justifying circumstance is invariably indicative of a conspiratorial agreement.
circumstance of self-defense, the presence of unlawful aggression is a
condition sine qua non. There can be no self-defense, complete or The rule is that where a conspiracy is proven, a showing as to who inflicted
incomplete, unless the victim has committed an unlawful aggression against the fatal wound is not required to sustain a conviction. The act of one in
the person defending himself. In the present case, the burden of evidence killing the victim becomes the act of all the accused.
having been shifted, we hold that the defense failed to establish the primary
element of unlawful aggression on the part of the victim and, therefore, the
plea of self-defense must fail. The narrations of the sequence of events by FACTS:
the accused, and by the lone alleged eyewitness for the defense, are Ernesto Romualdez was accosted by appellant near the barangay hall for
unconvincing primarily on account of their inherent inconsistency and allegedly circulating the rumor that appellant and his companions were
conflict with each other. engaged in stealing. Upon confrontation, appellant boxed Romualdez which
caused the latter to fall. Wilfredo Longno, who was then present at the
Qualifying Circumstances; Under such considerations and there being no scene, approached and helped the fallen Romualdez and pushed appellant
other evidence to prove that the death of the victim was the result of away. This apparently angered appellant who, in his native dialect said,
meditation, calculation or reflection, evident premeditation cannot be Andam ka lang Inday kay patyon ta guid, (Watch out Inday for I will kill
appreciated to qualify the killing to murder. The circumstances qualifying or you) to which Longno retorted, Just do it.
aggravating the act must be proved in an evident and incontestable manner.
They must be proved as conclusively as the acts constituting the offense. Two days later, appellant and his cousin, Cornelio Altejos, were drinking
Thus, for the same reason, the aggravating circumstance of abuse of softdrinks at the store of Gloria Aposaga when Longno passed by.
superior strength cannot be appreciated in this case. Superior strength may Thereupon, appellant and Altejos left their softdrinks half-consumed and
aggravate or qualify a crime, only if it is clearly shown that there was followed Longno who eventually reached the bench near the public faucet
deliberate intent to take advantage of it. Absence of any evidence to show where a group of guys were sitting and joined the group in their
conversation. Shortly thereafter, appellant and Altejos arrived and appellant
that the accused purposely sought to use their superior strength to their
accosted and pointed a gun at Longno, saying, Maano ka? (What are you
advantage in this case, a finding to that effect by the trial court cannot be going to do?). Longno then faced appellant and said, Brod, tiruha lang.
sustained. (Brod, just shoot.)
Apparently irked by the response, appellant fired the gun, hitting Longno in Facts:
the left forearm. Dullete, Canoso and Ramos then scampered for safety as
appellant and the wounded Longno grappled for the gun. It was while the As the deceased Barion and the defendant Alconga were playing black jack,
two were thus struggling that Altejos stabbed Longno in the chest, after the former became outraged and expressed his rage at the latter for his
which both appellant and Altejos ran away. cheating tactics. Two (2) days later, making good the threat uttered after the
game, Barion approached the defendant and swung his pingahan, a bamboo
Appellants version of the incident, however, differs. He admits having shot stick. A fight ensued afterwards, upon which Alconga gained the upper hand
Longno but pleads self-defense. He claims that Altejos for help to have a . by firing his revolver at Barion and eventually inflicting a mortal bolo blow,
22 caliber revolver repaired and appellant was taking the revolver to a slashing the latters cranium. The argument of self defense was raised by
policeman friend of his. On their way, appellant saw Longno from a Alconga.
distance. Upon his approach, Longno allegedly said, Insik, I heard that you
are not afraid of me. Maybe you want to be taught a lesson. Issues:

Appellant claims that the deceased had a revolver tucked in his waist and 1 Whether or not the Alcongas plea of self defense can be sustained
was about to draw the same. He, therefore, parried the gun but it fired 2 Whether or not Barions provocation is considered a mitigating
hitting one of appellants left fingers which was later amputated. It was then circumstance
that appellant pulled out his gun and shot Longno in the forearm. Appellant
and Longno afterwards grappled for the gun. Altejos allegedly tried to Ruling:
separate appellant and Longno but he was brushed aside by the latter. In the 1 NO.
course of their struggle, Altejos then shouted to appellant, I stabbed Inday,
run, and so he and Altejos ran away. There were two (2) stages in the fight- the first wherein the deceased
was the aggressor and the second wherein there is no more aggression,
since Barion was already injured and fleeing. Under such
circumstances, Alcongas plea of self-defense (as a justifying
ISSUE/S:
circumstance) during the second stage of the fight cannot be sustained.
1 Did the accused act in self-defense? There can be no defense where there is no aggression, given that the
2 Whether or not qualifying circumstances are present? aggression must be deemed to have ceased upon the flight of the
3 Is there a conspiracy between the accused and Altejos? deceased during the first stage. A fleeing man is not dangerous to the
one from whom he flees.

2 NO.
Note:
Section 4, Article 13 of the Revised Penal Code requires that there be
Insofar as Cornelio Altejos is concerned, however, the trial court never sufficient provocation or threat on the part of the offended party
acquired jurisdiction over him and he can neither be convicted nor immediately preceding the act. Sufficiency, as defined by Justice
exculpated herein. References in this judgment to him are, therefore, obiter Albert, means that it should be proportionate to the act committed and
and with no binding effect on him. adequate to stir one to its commission. It would seem self-evident that
Alconga could never have succeeded in showing that Barions
aggression was proportionate to his killing his already defeated
PEOPLE v. ALCONGA adversary. Also, it is noteworthy to mention that at Alconga, as a guard
by profession, was the holder of more deadly weapons and has superior (Benjamin), in turn, took another piece of stone, and hit Piol with it. This
fighting ability than Barion. would have been impossible, however, unless Benjamin first released the
dagger, which he then held; but, We cannot believe that he, or anybody for
THE PEOPLE OF THE PHILIPPINES vs. BENJAMIN GONDAYAO, that matter, would have done so under the circumstances.
alias "BEN", ET AL.
Again, when Piol allegedly squeezed the neck of Benjamin, the latter threw
Article 11. Justifying Circumstances- 1. Anyone who acts in defense of the stone away and picked up the dagger once more. He would thus have Us
his person or rights, provided that the following circumstances concur: believe that, in order to get the stone with which he claimed to have hit Piol
First. Unlawful aggression; Second. Reasonable necessity of the means on the head, he put the dagger in a convenient place from which, at the
employed to prevent or repel it; Third. Lack of sufficient provocation on the opportune moment, he got it back to inflict the second stab wound. The
part of the person defending himself. context of Benjamin's story does not convey the idea that he had such a
control of the situation as to be able to choose the place where he would put
Article 13. Mitigating Circumstance- 1. Those mentioned in the preceding the dagger and the time he would retrieve it.
chapter, when all the requisites necessary to justify the act or to exempt
from criminal liability in the respective cases are not attendant. But, this is not all. Instead of stabbing Piol on the stomach, for, by this time,
he was again lying down on his back according to the defense with
Article 14. Aggravating Circumstance- 16. That the act be committed Benjamin on top of him, he (Benjamin) thrust the dagger, with his left hand,
with treachery (alevosia). into the back of Piol, causing therein another stab wound almost at right
angle with his body, like the first. Just why, being in the precarious
There is treachery when the offender commits any of the crimes against the condition he depicted himself, Benjamin chose to stab Piol in such an
person, employing means, methods, or forms in the execution thereof which awkward, inconvenient and unbelievable manner, the defense has not even
tend directly and specially to insure its execution, without risk to himself tried to explain. Regardless of the foregoing, an injury inflicted in this
arising from the defense which the offended party might make. fashion on Piol's back, which was allegedly pressed against the ground,
would have necessarily been much more slanting than the first, instead of
No unlawful aggression when there is an agreement to fight. The challenge being almost perpendicular to the body. The fact of the matter and this
to a fight must be accepted. The reason for the rule is that each of the has been established by the testimony of the Chief of Police, whose
protagonists is at once assailant and assaulted and neither can invoke the impartiality and veracity are not contested is that Piol was then lying
right to self-defense because aggression which is incident in the fight is down, not on his back, but on his stomach with Benjamin on top of him.
bound to arise from one or the other of the combatants. This explains why and how he (Benjamin) managed to stab Piol on the
back. It, likewise, shows that Piol could not have struck Benjamin on the
Unlawful aggression must come, directly or indirectly, from the person who face with a stone, much less squeezed his neck.
was subsequently attacked by the accused.
Benjamin stabbed Piol twice from behind, after disarming him.
RULING: Benjamins story is manifestly artificious and unworthy of Considering, moreover, that Benjamin had provoked the incident, by
credence. It should be noted that, according to Benjamin, he held the hurling uncomplimentary remarks at his political opponents, one of whom
dagger, even before they fell from the "papag"; that the dagger was still in was Piol; that such remarks led to an altercation with Piol, in consequence
his hand when Piol allegedly struck his face with a stone; and that he of which, stones were thrown at him, hitting him on the head; that when,
owing to the impact of said stone, which could have rendered him groggy, Benjamin testified that his remark, about the failure to give a blow-out on
and the lacerated injuries thus sustained by him, Piol prepared himself to the part of the winner in the elections, was addressed to his nephew, Rudy
fight by drawing out his dagger, Benjamin accepted the challenge resulting Natividad; that, when Piol resented said remark, Benjamin replied that the
from this act, by "rushing" to his encounter and grappling with him; and same was not aimed at him, and that, perhaps, he (Piol) is a "nacionalista".
that, accordingly, Benjamin cannot be given the benefit of either complete
or incomplete self-defense. Although Piol was stabbed from behind, ISSUE: Whether or not the accused acted in self-defense.
Benjamin did not act with treachery, for this was merely an incident of their
struggle, which had begun with both contenders facing each other, each G.R. No. L-28129 October 31, 1969
prepared for the fight that ensued.
ELIAS VALCORZA, petitioner,
The crime committed by Benjamin Gondayao is, therefore, that of vs.
homicide, and no modifying circumstance having attended its perpetration, PEOPLE OF THE PHILIPPINES, respondent.
the penalty therefor (reclusion temporal) should be imposed in its medium
period. Pursuant to the Indeterminate Sentence Law, he should, accordingly,
be sentenced to an indeterminate penalty ranging from 8 years and 1 day
of prison mayor, as minimum, to 14 years, 8 months and 1 day of reclusion RULING
temporal as maximum, with the corresponding accessory penalties.
The petitioner is acquitted.
FACTS: Appeal, taken by defendants Benjamin Gondayao and Anoy
Gondayao from a decision convicting them of the crime of murder of
Orlando Piol, qualified by treachery. ART. 11. Justifying circumstances. - The following do not incur any
criminal liability
After consuming a bottle of gin, Benjamin, repeatedly cursed the Paragraph 5. Any person who acts in the fulfillment of a duty or in
"Nacionalistas" for not offering a drink despite their victory in the polls; the lawful exercise of a right or office.
that, as Piol who was nearby, replied by cursing the losers in the elections,
an exchange of unfriendly utterances followed; that, eventually, the group
headed by Benjamin picked up stones and threw them at Piol, who was hit
several times on the head; that, as the same began to bleed, Piol drew out a The facts and circumstances constrain us to hold that the act thus performed
dagger and approached Benjamin, who, forthwith, embraced him; that, by petitioner, which unfortunately resulted in the death of the escaping
grappling with each other, they both fell down. detainee, was committed in the performance of his official duty and was
more or less necessary to prevent the escaping prisoner from successfully
eluding the officers of the law. To hold him guilty of homicide may have the
After grappling with Benjamin Gondayao, Orlando Piol appeared to have
effect of demoralizing police officers discharging official functions identical
in addition to several lacerations on the head two (2) stab wounds on
or similar to those in the performance of which petitioner was engaged at
the back, 4 to 4-1/2 inches deep, in consequence of which he died due to a
the time he fired at the deceased Pimentel, with the result that thereafter We
massive internal hemorrhage. Immediately after the occurrence, Benjamin
would have half-hearted and dispirited efforts on their part to comply with
assumed full responsibility for said injuries, which he claimed to have
such official duty. This of course, would be to the great detriment of public
inflicted in self-defense.
interest.
motive was true only insofar as Avelino was concerned. The circumstances
indicate that if Jose embraced Rodrigo and rendered him helpless, it was to
FACTS: stop him from further hitting Avelino with his fists. However, Jose is not
The deceased, Roberto Pimentel was a detention prisoner who escaped. entirely free from liability, for it has been established that even after the first
While in search for the escaped prisoner, Sgt. Daiton saw a person knife thrust had been delivered he did not try to stop Avelino, either by
approaching slowly under the bridge and he ordered him to halt. The latter word or overt act. Instead Jose continued to hold Rodrigo, even forced him
instead of doing so, jumped down into the creek spanned by the down on the bamboo bed with Avelino still pressing the attack. Withal it
bridge. Roberto Pimentel emerged suddenly from the bushes and lunged at cannot be said that Jose's cooperation was such that without it the offense
the appellant Valcorza, hitting him with a stone and causing him to fall to would not have been accomplished. But although not indispensable, it was a
the ground. Appellant Valcorza regained his composure and immediately contributing factor. If Jose's initial intent was free from guilt, it became
chased the deceased, as the deceased did not heed his order to stop, tainted after he saw the first knife thrust delivered. The thirteen wounds
appellant fired four times into the air, and a fifth shot at Pimentel as the must have taken an appreciable interval of time to inflict, and Jose's
latter was in an act of again jumping down into another part of the creek. cooperation facilitated their infliction. He must therefore be held liable as
The members of the patrol team went down into the water to locate an accomplice.
Pimentel and they saw him floating, with a wound on his back. Elias
Valcorza surrendered himself and his firearm to the Chief of Police
Facts: Avelino was eating his lunch inside one of the eateries dotting the
market site, when Rodrigo Aringo alias Diego, a baggage boy in the same
ISSUE: market, approached him and demanded his fee for having carried Avelino's
baggage. When Avelino said he was willing to pay for the services rendered
Whether or not peace officer Valcorza was justified in shooting the deceased at noon, but not for those rendered earlier in the morning, Rodrigo
brusquely brushed Avelino's hand aside and instantly gave him a fist blow
in the face. A quarrel between them ensued and Jose Manansala, Avelino's
PEOPLE OF THE PHILIPPINES vs. AVELINO MANANSALA, JR. uncle, noticed the commotion and so he went there and had Rodrigo in a
and JOSE MANANSALA tight embrace. While in that position, Avelino stabbed him with a balisong,
or Batangas knife thirteen times which resulted to Rodrigo's death. Avelino
G.R. No. L-23514, February 17, 1970 and his uncle, Jose were charged with murder and Jose was considered as a
co-principal by the CFI.

Article 17 of the RPC; Article 18 of the RPC states that Accomplices are
those persons who, not being included in Art.17, cooperate in the execution Issue: Whether or not Jose Manansala was correctly considered as a co-
of the offense by previous or simultaneous acts. In this case, Jose principal by the CFI?
Manansala was found guilty as co-principal on the ground that there was
concert of action between him and his nephew. The evidence does not
justify this finding beyond reasonable doubt. There is no showing that the Case #32.
killing was agreed upon between them beforehand. No motive for it has
been shown other than the provocation given by the deceased; and such People of the Philippines vs Jose Encomienda
GR No L-26750; August 18, 1972 It was the deceased who drew his gun towards the accused. Not
having enough time to think rationally how to deal with the situation, the
accuseds act of hacking the left arm of deceased is justified due to
Article 11, Revised Penal Code/ Legitimate Self-Defense, elements: (1) immediate danger to his life as the former was about to grab gun from his
unlawful aggression on part of the victim; (2) reasonable necessity of the free left hand. Provocation came from the deceased (not the accused) as he
means, employed to prevent or repel the attack; and (3) lack of sufficient ordered the accused to vacate land and drew his gun when he did not like
provocation on the part of the person defending himself the response of the former.

Illegal Aggression, what constitutes: Illegal aggression is People vs Sotelo


equivalent to assault or at least threatened assault of immediate and 55 PHIL 403
imminent kind.
December 13, 1930
Reasonable Necessity of the Means Employed, what
constitutes: Reasonable necessity of the means employed does not
imply material commensurability between the means of attack and Article 13 of the RPC: Incomplete Self Defense
defense. When the law requires is rational equivalence, in the
consideration of which will enter as principal factors the
emergency, the imminent danger to which the person attacked is Facts:
exposed and the instinct, more than the reason, that moves or
The Sotelo brothers, namely, Constante, Dominador, and Vicente, were
impels the defense, and the proportionateness thereof does not
prosecuted in the Court of First Instance of Ilocos Sur for the crime of
depend upon the harm done, but rests upon the imminent danger of homicide. The said accused conspired to attack Ignacio Cambaliza and
such injury. inflicted a mortal wound upon him which he died in a few minutes
afterwards.

Facts: Accused was charged with the crime of murder aggravated by Issue:
recidivism. He interposed the lawful self-defense as per Article 11 of the Whether or not the accused Constante can invoke self defense?
Revised Penal Code.

Issue: Whether or not the accused satisfied the elements of lawful self-
defense? Ruling:
Held: Yes, the accused sufficiently proved the elements for lawful self- The selfdefense invoked by the accused was incomplete, for, although the
defense. Elements are: (1) unlawful aggression on part of the victim; (2) appellant was unlawfully attacked by the deceased and compelled to
reasonable necessity of the means, employed to prevent or repel the attack; employ reasonable means to defend himself, he is responsible for provoking
and (3) lack of sufficient provocation on the part of the person defending the attack.
himself.
PEOPLE vs. HERNANDEZ
GR NO. L-6025, MAY 30 1964 The most important activity of appellant Hernandez appears to be the
Labrador, J.: propagation of improvement of conditions of labor through his organization,
the CLO. While the CLO of which he is the founder and active president,
Article 136. Conspiracy and proposal to commit rebellion or insurrection has communistic tendencies, its activity refers to the strengthening of the
unity and cooperation between labor elements and preparing them for
FACTS: struggle; they are not yet indoctrinated in the need of an actual war with or
against Capitalism. The appellant was a politician and a labor leader and it
About March 15, 1945, Amado Hernandez and other appellants were is not unreasonable to suspect that his labor activities especially in
accused of conspiring, confederating and cooperating with each other, as connection with the CLO and other trade unions, were impelled and
well as with the thirty-one (31) defendants charged in the criminal cases of fostered by the desire to secure the labor vote to support his political
the Court of First Instance of Manila. They were accused of being members ambitions. It is doubtful whether his desire to foster the labor union of
of PKP Community Party of the Philippines which was actively engaged in which he was the head was impelled by an actual desire to advance the
an armed rebellion against the government of the Philippines. With the cause of Communism, not merely to advance his political aspirations.
party of HUKBALAHAP (Hukbo ng Bayan Laban sa mga Hapon), they
committed the crime of rebellion causing murder, pillage, looting plunder, Insofar as the appellant's alleged activities as a Communist are concerned,
etc., enumerated in 13 attacks on government forces or civilians by HUKS. We have not found, nor has any particular act on his part been pointed to
Us, which would indicate that he had advocated action or the use of force in
ISSUE: Does his or anyones membership in the communist party per se securing the ends of Communism. True it is, he had friends among the
render Hernandez or any Communist guilty of conspiracy to commit leaders of the Communist Party, and especially the heads of the rebellion,
rebellion under the provisions of Article 136 of the RPC? but this notwithstanding, evidence is wanting to show that he ever attended
their meetings, or collaborated and conspired with said leaders in planning
HELD: and encouraging the acts of rebellion, or advancing the cause thereof.
Insofar as the furnishing of the mimeograph machine and clothes is
No. The advocacy of Communism or Communistic theory and principle is concerned, it appears that he acted merely as an intermediary, who passed
not to be considered as a criminal act of conspiracy unless transformed or said machine and clothes on to others. It does not appear that he himself
converted into an advocacy of action. In the very nature of things, mere furnished funds or material help of his own to the members of the rebellion
advocacy of a theory or principle is insufficient unless the communist or to the forces of the rebellion in the field.
advocates action, immediate and positive, the actual agreement to start an
uprising or rebellion or an agreement forged to use force and violence in an But the very act or conduct of his in refusing to go underground, in spite of
uprising of the working class to overthrow constituted authority and seize the apparent desire of the chief of the rebellion, is clear proof of his non-
the reins of Government itself. Unless action is actually advocated or participation in the conspiracy to engage in or to foster the rebellion or the
intended or contemplated, the Communist is a mere theorist, merely holding uprising.
belief in the supremacy of the proletariat a Communist does not yet
advocate the seizing of the reins of Government by it. As a theorist the
Communist is not yet actually considered as engaging in the criminal field
subject to punishment. Only when the Communist advocates action and
actual uprising, war or otherwise, does he become guilty of conspiracy to G.R. No. L-4445 February 28, 1955
commit rebellion.
THE PEOPLE OF THE PHILIPPINES, plaintiff-appellee,
vs.
MANUEL BERONILLA, FILIPINO VELASCO, POLICARPIO indicted in the Court of First Instance of Abra for murder, for allegedly
PACULDO, and JACINTO ADRIATICO, defendants-appellants conspiring and confederating in the execution of Arsenio Borjal.

FACTS:

Arsenio Borjal was the elected mayor of La Paz, Abra, at the outbreak of ISSUE:
war, and continued to serve as Mayor during the Japanese occupation, until
March 10, 1943, when he moved to Bangued because of an attempt upon whether or not this message, originally sent to Arnold's quarters in San
his life by unknown persons. On December 18, 1944, appellant Manuel Esteban, Ilocos Sur, was relayed by the latter to appellant Beronilla in La
Beronilla was appointed Military Mayor of La Paz by Lt. Col. R. H. Arnold, Paz, Abra, on the morning of April 18, 1945, together with the package of
regimental commander of the 15th Infantry, Philippine Army, operating as a records of Borjal's trial that was admittedly returned to and received by
guerrilla unit in the province of Abra. Beronilla on that date, after review thereof by Arnold (Exhibit 8-8-a).

RULING:

Sometime in March, 1945, while the operations for the liberation of the
province of Abra were in progress, Arsenio Borjal returned to La Paz with We have carefully examined the evidence on this important issue, and find
his family in order to escape the bombing of Bangued. Beronilla, pursuant no satisfactory proof that Beronilla did actually receive the radiogram
to his instructions, placed Borjal under custody and asked the residents of Exhibit H or any copy thereof. The accused roundly denied it.
La Paz to file complaints against him. In no time, charges of espionage,
aiding the enemy, and abuse of authority were filed against Borjal.
. In addition to Balmaceda was contradicted by Bayken, another prosecution
witness, as to the hatching of the alleged conspiracy to kill Borjal.
The trial lasted 19 days up to April 10, 1945; the jury found Borjal guilty on Balmaceda claimed that the accused-appellants decided to kill Borjal in the
all accounts and imposed upon him instruction from his superiors. And on early evening of April 18, while Bayken testified that the agreement was
the night of the same day, April 18, 1945, Beronilla ordered the execution of made about ten o'clock in the morning, shortly after the accused had denied
Borjal. Jacinto Adriatico acted as executioner and Antonio Palope as grave Borjal's petition to be allowed to hear mass.
digger.
Our conclusion is that Lt. Col. Arnold, for some reason that can not now be
ascertained, failed to transmit the Volckmann message to Beronilla. And this
Two years thereafter, Manuel Beronilla as military mayor, Policarpio being so, the charge of criminal conspiracy to do away with Borjal must be
Paculdo as Clerk of the jury, Felix Alverne and Juan Balmaceda as rejected, because the accused had no need to conspire against a man who
prosecutors, Jesus Labuguen, Delfin Labuguen, Filemon Labuguen, was, to their knowledge, duly sentenced to death.
Servillano Afos, Andres Afos, Benjamin Adriatico, Juanito Casel, Santiago
Casel, Mariano Ajel, Felix Murphy, Benjamin Abella, and Pedro Turqueza
as members of the jury, Jacinto Adriatico as executioner, Severo Afos as
grave digger, and Father Filipino Velasco as an alleged conspirator, were decided that the concurrence of personal hatred and collaboration with the
enemy as motives for a liquidation does not operate to exclude the case
from the benefits of the Amnesty claimed by appellants, since then "it may
not be held that the manslaughter stemmed from purely personal motives" FACTS:
Actually, the conduct of the appellants does not dispose that these appellants
were impelled by malice. The accused PO3 Ferdinand Fallorina y Fernando, with intent to kill, by
means of treachery and taking advantage of superior strength, did then and
The lower Court, after finding that the late Arsenio Borjal had really there, wilfully, unlawfully and feloniously attack, assault and employ
committed treasonable acts, (causing soldiers and civilians to be tortured, personal violence upon the person of VINCENT JOROJORO, JR. y
and hidden American officers to be captured by the Japanese) expressly MORADAS, a minor, eleven (11) years of age, by then and there, shooting
declared that "the Court is convinced that it was not for political or personal him with a gun, hitting him on the head, thereby inflicting upon him serious
reason that the accused decided to kill Arsenio Borjal" and mortal wound which was the direct and immediate cause of his death,
to the damage and prejudice of the heirs of the said offended party.

The trial court rendered judgment convicting the appellant of murder,


Actus non facit reum nisi mens si rea. qualified by treachery and aggravated by abuse of public position. The trial
court did not appreciate in favor of the appellant the mitigating
To constitute a crime, the act must, except in certain crimes made circumstance of voluntary surrender.
such by statute, be accompanied by a criminal intent, or by such
ISSUE:
negligence or indifference to duty or to consequence, as, in law, is
equivalent to criminal intent. The maxim is, actus non facit reum, Whether or not the appellant is exempt from criminal liability.
nisi mens rea-a crime is not committed if the minds of the person
performing the act complained of be innocent. (U. S. vs. Catolico,
18 Phil., 507).
RULING:

The appellant was burdened to prove, with clear and convincing evidence,
G.R. No. 137347 March 4, 2004 his affirmative defense that the victim's death was caused by his gun
accidentally going off, the bullet hitting the victim without his fault or
intention of causing it; hence, is exempt from criminal liability under
PEOPLE OF THE PHILIPPINES, appellee,
Article 12, paragraph 4 of the Revised Penal Code which reads
vs.
PO3 FERDINAND FALLORINA Y FERNANDO, appellant.
The following are exempt from criminal liability:


Article 248 of the Revised Penal Code, the penalty for murder is
reclusion perpetua to death. Since there is no modifying circumstance 4. Any person who, while performing a lawful act with due care,
in the commission of the crime, the appellant should be sentenced to causes an injury by mere accident without fault or intention of
suffer the penalty of reclusion perpetua, conformably to Article 63 of the causing it.
Revised Penal Code.
The basis for the exemption is the complete absence of intent and
negligence on the part of the accused. For the accused to be guilty of a
felony, it must be committed either with criminal intent or with fault or defenseless position of Chua when he was killed, while nighttime was
negligence.33 purposely sought by the accused to facilitate immunity in the commission of
the crime. The aggravating circumstance of uninhabited place (despoblado)
The elements of this exempting circumstance are (1) a person is performing is also present, due to the deliberate selection of an isolated place (Barrio
a lawful act; (2) with due care; (3) he causes an injury to another by mere Makatipo Novaliches, Caloocan City) for killing and burying the victim
accident; and (4) without any fault or intention of causing it. 34 An accident
The aggravating circumstance of use of motor vehicle in the commission of
is an occurrence that "happens outside the sway of our will, and although it the crimes," can be considered present because the Biscayne car of Ong was
comes about through some act of our will, lies beyond the bounds of used to trail the victim's car and to facilitate the commission of the crimes,"
humanly foreseeable consequences." If the consequences are plainly and the escape of the accused.
foreseeable, it will be a case of negligence.
Evident premeditation attended the commission of the crimes, because the
The appellant committed murder under Article 248 of the Revised Penal accused meditated, planned, and tenaciously persisted in the
Code qualified by treachery. accomplishment of the crime.

Under Article 248 of the Revised Penal Code, the penalty for murder is Accused Ong was given the mitigating circumstances of plea of guilty and
one analogous to passion and obfuscation" because Chua previously
reclusion perpetua to death. Since there is no modifying circumstance in the
threatened Ong for non-payment of debt arising from gambling, causing
commission of the crime, the appellant should be sentenced to suffer the
Ong humiliation and shame.
penalty of reclusion perpetua, conformably to Article 63 of the Revised "Conspiracy, connivance and unity of purpose and intention among the
Penal Code. accused were present throughout in the execution of this crime. The four
participated in the planning and execution of the crimes," and were at the
People VS. Ong scene in all its stages. They cannot escape the consequences of any of their
acts even if they deviated in some detail from what they originally thought
of. Conspiracy implies concert of design and not participation in every
detail of the execution. Thus, treachery should be considered against all
Intro:
persons participating or cooperating in the perpetration of the crime."
All contrary to law with the following generic aggravating circumstances:
(a) Evident premeditations;
Brief Facts:
(b) Grave abuse of confidence;
That on or about April 23 to April 24, 1971, inclusive, in the municipality of
(c) Nighttime;
Paraaque, province of Rizal, Philippines, and within the jurisdiction of this
(d) Use of an motor vehicle ;
Honorable court, the above named accused, being then private individuals,
(e) Use of uperior strenght;
conspiring and confederating together and mutually helping one another,
(f) Cruelty.
did then and there wilfully, unlawfully and with treachery and known
premeditation and for the purpose of killing one Henry Chua and thereafter
Kidanapping with Murder as defined under Article 248 of the Revised Penal
extorting money from his family through the use of a ransom note,
Code, in relation to Article 267
kidnapped and carried away said Henry Chua, initially by means of friendly
gestures and later through the use of force, in an automobile, and later after
The aggravating circumstance of abuse of superior strength is absorbed in
having taken him to an uninhabited place in Caloocan City, with the use of
treachery. The aggravating circumstance of nighttime (nocturnidad) cannot
force detained him (Henry Chua) and killed him in the following manner, to
be absorbed in treachery because in this crimes," treachery arose from the
wit: The accused after gagging and tying up Henry Chua and repeatedly (b) an act manifestly indicating that the accused clung to that
threatening him with death, assured him that if he would write and sign a determination, and
ransom note for the payment by his family of the sum of $50,000.00 (US),
he would not be killed and would be released upon receipt of the ransom (c) a lapse of time between the determination and the execution
note, he was again gagged and tied up by the accused, and thereafter sufficient to allow the accused to reflect upon the consequences of
stabbed in the abdominal region, several times with an ice-pick, inflicting
the act
upon him (Henry Chua) mortal wounds on his vital organs, which directly
caused his death.
In the case at bar, the prosecution fails to prove this aggravating
Brief Issue: Circumstance

The only issue in this case, therefore, is whether or not the accused II. Cruelty
Ambrosio voluntarily participated in the commission of the crime.
For cruelty to be appreciated against the accused, it must be shown that the
accused, for his pleasure and satisfaction, caused the victim to suffer slowly
Case #41 and painfully as he inflicted on him unnecessary physical and moral pain.

Rationale: The crime is aggravated because by deliberately increasing the


suffering of the victim the offender denotes sadism and
PEOPLE OF THE PHILIPPINES, plaintiff-appellee, vs. GEORGE consequently a marked degree of malice and perversity.
CORTES y ORTEGA, accused-appellant. [G.R. No. 137050. July 11, 2001]
In the case at bar, The mere fact of inflicting various successive wounds
upon a person in order to cause his death, no appreciable time intervening
between the infliction of one (1) wound and that of another to show that he
*in this case the court took little discussions on the application of the following had wanted to prolong the suffering of his victim, is not sufficient for taking
aggravating circumstances as alleged in the complaint. this aggravating circumstance into consideration.

Nighttime

I Evident Premeditation. Night-time becomes an aggravating circumstance only when:

In the aggravating circumstance of evident premeditation, the prosecution (1) it is specially sought by the offender;
must prove the following elements:
(2) the offender takes advantage of it; or
(a) the time when the accused determined to commit the crime,
(3) it facilitates the commission of the crime by insuring the offender's
immunity from identification or capture.
In the case at bar, no evidence suggests that accused purposely sought the FACTS:
cover of darkness to perpetrate the crime, or to conceal his identity.
1 an Information for murder (violation of Article 248 of the Revised Penal
IV Abuse of Superior Strength Code) against accused On George Cortes y Ortega, for the murder of Edlyn
S. Gamboa, a 16 year old girl
Abuse of superior strength is absorbed in treachery, so that it cannot be
appreciated separately as another aggravating circumstance. Here, treachery 2 Accused admitted that he stabbed Edlyn.
qualified the offense to murder.
3 He entered a plea of guilty In virtue of his plea of guilty, the trial court
V Disregard of Sex proceeded to satisfy itself of the voluntariness of the plea by propounding
questions to the accused to find out if he understood his plea and the legal
As to the aggravating circumstance of disregard of sex, the same could not consequence thereof.
be considered as it was not shown that accused deliberately intended to
offend or insult the sex of the victim, or showed manifest disrespect for her 4 The prosecution alleged that the aggravating circumstances of evident
womanhood. premeditation, cruelty, nighttime, abuse of superior strength, disrespect to
sex, and intoxication were present in the commission of the crime.
In the case at bar, the accused mistook the victim for a man
5 The accused, on the other hand, raised the attendance of the mitigating
VI Intoxication circumstances of voluntary surrender, plea of guilty, mistaken identity and
the alternative mitigating circumstance of intoxication.
Ordinarily, intoxication may be considered either aggravating or mitigating,
depending upon the circumstances attending the commission of the crime. 6 the trial court after considering the aggravating and mitigating
circumstances attendant found the existence of the aggravating
Intoxication has the effect of decreasing the penalty, if it is not habitual or circumstances and appreciated only the mitigating circumstance of plea of
subsequent to the plan to commit the contemplated crime; on the other guilty that was offset by one of the aggravating circumstances. Hence, this
hand, when it is habitual or intentional, it is considered an aggravating review
circumstance.

A person pleading intoxication to mitigate penalty must present proof of


having taken a quantity of alcoholic beverage prior to the commission of the ISSUES:
crime, sufficient to produce the effect of obfuscating reason. At the same
time, that person must show proof of not being a habitual drinker and not Whether the trial court erred in finding that the aggravating
taking the alcoholic drink with the intention to reinforce his resolve to circumstances of evident premeditation, cruelty, nighttime, abuse of
commit the crime superior strength, sex and intoxication attended the commission of the crime
charged.
Ulanday, a health officer of Aroroy, testified herself that the complaining
witness either voluntarily submitted to a sexual act or was forced into
HELD: one.

The Solicitor General agrees with the accused that the only aggravating ISSUE:
circumstance present was treachery which qualified the killing to murder (a) Whether additional rape committed in a crime of robbery be considered
and that there were two mitigating circumstances of plea of guilty and as an aggravating circumstance?
intoxication, not habitual. The penalty shall be reclusion perpetua, not
death, in accordance with Article 63 in relation to Article 248 of the Revised
Penal Code, as amended by Republic Act No. 6759
HELD:
On cross-examination, both Nerissa Tagala and Consuelo Arevalo,
separately testified that they saw the face of Regala, despite of no electricity
CASE No. 42 at the commission of the crime, because he used a flashlight and took off
the mask he was wearing, and thus, they remembered him wearing an
People vs Regala earring of his left ear, which he was still wearing at the time of the police
G.R. No. 130508 April 5, 2000 line-up inside the police station.

The trial court held that contradiction referred to a minor detail, cannot
FACTS:
detract from the fact, that both Nerissa and Consuelo positively identified
On the night of September 11, 1995, at Barangay Bangon in Aroroy,
the accused-appellant. As correctly pointed out by the appellee, the victim
Masbate, then 16-year old victim Nerissa Tagala and her grandmother,
was a 16-year old barrio lass, not exposed to the ways of the world and was
Counselo Arevalo, were sleeping, when appellant Armando Regala and his
not shown to have any ill-motive to falsely implicate accused-appellant,
two other companions entered the formers house.
who was a stranger. Hence, Dr. Ulandays testimony does not support the
contention of accused-appellant that the victim voluntarily submitted to
Appellant and his companions entered the house through the kitchen and
sexual advances of Regala.
went to the room of the victims and poked at 8-inch gun on them, one after
the other, and hogtied both of them. Armando raped Nerissa in bed while
The crime of robbery with rape was committed in 1995 when RA 7659 was
her grandmother was hogtied on the floor. Later, she saw her grandmothers
already in force. Under Article 294 of the Revised Penal Code as amended,
aparador being opened where two rings, two wrist watches, and money
now provides, under paragraph 1 thereof: (1) The penalty of reclusion
were taken from the aparador. After raping her in bed, Nerissa saw accused-
perpetua to death, when for any reason of or on occasion of the robbery, the
appellant counting the money taken from the aparador. Thereafter, she was
crime of homicide shall have been committed, or when the robbery shall
brought to the kitchen, still hogtied and was raped again by the accused.
have been accompanied by rape or intentional mutilation or arson.
He was convicted in the lower court but accused-appellant appealed his
In this case, the additional rape committed by herein accused-appellant
criminal case at the Regional Trial Court in Masbate. He questioned the
should not be considered as aggravating. The penalty of reclusion perpetua
sufficiency of the prosecutions evidence in identifying him as one of the
imposed by the trial court is proper. The judgment convicting Armando
perpetrators of the crime charged. And based on medico-legal, Dr. Conchita
Regala y Abriol guilty beyond reasonable doubt of the crime of Robbery
with Rape, where the victim is entitled to an additional award of P50,000.00 store and were on their way to each of their homes, when a group of men
as civil indemnity. armed with bolos stopped them and introduced themselves as policemen,
PEOPLE vs BERDIDA saying that the four are accused of killing someone and that they should go
GR No. L-20183; June 30, 1966 with them. Antonio and Federico held their hands up while Virgilio and
Pedrito could ran away. Antonio and Federico were taken to a rail track
DOCTRINE:
where their hands were tied. They were then further taken to a pier in North
Rule of aggravating circumstances with regards to penalty the presence of
Harbor where they were met by more armed men. The group of men beat
one generic aggravating circumstance, apart from the qualifying
Antonio and Federico until Antonio lost consciousness at around 1:00am of
circumstance of treachery, suffices to fix the penalty for murder at the
the following day. The police, together with Antonio's sister, found Antonio
extreme punishment of death.
and Federico the following day. Antonio was still alive thus was rushed to
Nighttime from the facts and evidence of record in this case, appellants
the hospital while Federico was already dead.
took advantage of nighttime in committing the felonies charged for they had
evidently chosen to execute their victims under cover of darkness, at the ISSUE:
dead of night, when the neighborhood was asleep. 1. Was the RTC correct in imposing the death penalty?
Exception to the rule of absorbing Nighttime in Treachery in as much as 2. Was the appreciation of the aggravating circumstances proper?
the treachery consisted in the fact that the victims' hands were tied at the
time they were beaten, the circumstance of nighttime is not absorbed in
treachery, but can be perceived distinctly therefrom, since the treachery
People vs Castillo
rests upon an independent factual basis. A special case therefore is present
PEOPLE OF THE PHILIPPINES, appellee, vs. ELIZABETH
to which the rule that nighttime is absorbed in treachery does not apply. CASTILLO and EVANGELINE PADAYHAG, appellants.
Evident Premeditation from the time the accused abducted the victims up
[G.R. No. 132895. March 10, 2004]
to the time one of the latter lost consciousness and the other died, sufficient
Doctrine/s and/or Rulings: We affirm the trial courts judgment convicting
time had lapsed for the accused to meditate and reflect on the consequences
Castillo. However, we acquit her co-accused Padayhag.
of their act.
To sustain a conviction for Kidnapping and Serious Illegal Detention under
Article 267 of the Revised Penal Code,[8] the prosecution must establish
FACTS: the following: (1) the offender is a private individual; (2) he kidnaps or
At around 10:00pm, Antonio, Federico, Virgilio and Pedrito had just left a detains another or in any other manner deprives the victim of his liberty; (3)
the act of kidnapping or detention is illegal; and (4) in the commission of
the offense any of the following circumstances is present: (a) the There must be positive and conclusive evidence that Padayhag acted in
kidnapping or detention lasts for more than three days; (b) it is committed concert with Castillo to commit the same criminal act. To hold an accused
by simulating public authority; (c) serious physical injuries are inflicted on guilty as a co-principal by conspiracy, there must be a sufficient and
the victim or threats to kill are made; or (d) the person kidnapped or unbroken chain of events that directly and definitely links the accused to the
detained is a minor, female or a public officer.[9] commission of the crime without any space for baseless suppositions or
frenzied theories to filter through.[26] Indeed, conspiracy must be proven as
Appellant Castillos Liability clearly as the commission of the crime itself.[27]
Castillo asserts that the victims parents did not pay her wages when she Conspiracy is established by the presence of two factors: (1) singularity of
worked as a maid of the victims family.[10] She claims that it was this intent; and (2) unity in execution of an unlawful objective. The two must
injustice, her educational level and her ignorance of the law, which impelled concur. Performance of an act that contributes to the goal of another is not
her to take Rocky. She faults the trial court for refusing to consider this. enough. The act must be motivated by the same unlawful intent. Neither
Castillo is mistaken. Whether or not her employer failed to pay her salary is joint nor simultaneous action is per se sufficient indicium of conspiracy,
irrelevant. No amount of perceived injustice can serve as justification for unless proved to have been motivated by a common design.[28]
any person to retaliate through the commission of another crime. The trial
court was therefore correct in disregarding Castillos claim that Rockys Padayhags act of fetching Rocky is not conclusive proof of her complicity
parents committed injustice on her. with Castillos plan, a plan Padayhag did not even know. Both appellants
testified that Padayhag met Castillo only because Castillo told Padayhag
Castillos claim of injustice cannot justify in any way her demand for that Padayhags boyfriend was sick. It was precisely on the pretext that they
ransom. Ransom is money, price or consideration paid or demanded for were to visit Padayhags boyfriend that the two met. When they met,
redemption of a captured person or persons, a payment that releases from Padayhag realized that Castillo had deceived her:
captivity.[11] Thus, even if she had a right to demand payment of her
unpaid wages, the money she actually demanded and eventually received, is All these circumstances illustrate the absence of any hint of conspiracy. We
still ransom. also find that the prosecution failed to prove Padayhags guilt beyond
reasonable doubt. In People v. Gonzales[36] we held:
Castillos reliance on her low educational level is similarly unavailing. The
penalty for kidnapping for ransom is the singular and indivisible penalty of In the absence of conspiracy, if the inculpatory facts and circumstances are
death. This bars the application of any alternative, mitigating or aggravating capable of two or more explanations, one of which is consistent with the
circumstance innocence of the accused and the other consistent with his guilt, then the
evidence does not fulfill the test of moral certainty and is not sufficient to
Appellant Padayhags Liability support a conviction.
The same cannot be said of Padayhag. Our review of the evidence on record Penalty and Damages
shows that the prosecution failed to prove Padayhags guilt beyond
reasonable doubt. Under Article 267 of the Revised Penal Code,[44] the penalty of death is
imposed upon proof that the kidnapping was committed to extort ransom
We reiterate the doctrine that an appeal in a criminal case opens the entire from the victim or any other person. We find that the prosecution has
case for review on any question including those not raised by the parties. established Castillos guilt for this crime beyond reasonable doubt. However,
[25] This becomes even more imperative in cases where the penalty Castillos pecuniary liability must be modified to conform with
imposed is death. jurisprudence. The award of exemplary damages must be deleted in the
absence of any aggravating circumstance. Mr. Cebrero testified that their
family suffered serious anxiety at the possibility of not seeing Rocky again.
[45] The pain and anguish they experienced justifies the award of moral People vs Tan, G.R. No. 132324. September 28, 1999
damages. However, we reduce the trial courts award of moral damages to
Doctrines:
P100,000 in line with current jurisprudence.
Self defense
Brief Facts: Before us on automatic review is the Decision[1] of the
Regional Trial Court of Paranaque, Branch 260, National Capital Judicial When the accused invoke self-defense, the burden of proof is shifted to
Region, in Criminal Case No. 95-86, finding appellants Elizabeth Castillo them to prove that the killing was justified and that they incurred no
(Castillo) and Evangeline Padayhag (Padayhag) guilty of Qualified criminal liability therefor. They must rely on the strength of their own
Kidnapping and Serious Illegal Detention[2] and sentencing them to death. evidence and not on the weakness of that of the prosecution, for even if the
latter is weak, it could not be disbelieved after their open admission of
The Information[3] charging Castillo, Padayhag and Imelda Wenceslao with responsibility for the killing.
the crime of kidnapping, reads:
In the present case, it is incumbent upon Appellant Norly Tan to prove self-
That on or about March 1, 1995, in Paraaque, Metro Manila, Philippines, defense. Thus, he must prove that there was unlawful aggression on the part
and within the jurisdiction of the Honorable Court, said accused of the victim, that the means employed to prevent it were reasonable, and
ELIZABETH CASTILLO and EVANGELINE PADAYHAG, conspiring that there was lack of sufficient provocation o his part. However, he failed
to discharge this burden.
together, confederating, and mutually helping one another, did then and
there willfully, unlawfully and feloniously kidnap, carry away, and seriously
detain HORACIO CEBRERO IV @ Rocky, a five years old child (sic),
Crime and its Punishment
which kidnapping or serious detention lasted for more than three (3) days
thereby depriving him of his liberty, and which was committed for the The killing was attended by treachery; hence, the crime was murder. The
purpose of extorting ransom from the parents of the victim, to the damage essence of treachery is the sudden and unexpected attack, without the
and prejudice of the victim himself and his parents. slightest provocation on the part of the person attacked. Treachery is present
when the offender commits any of the crimes against persons, employing
Issues: means, methods or forms in the execution thereof, which tend directly and
especially to insure its execution, without risk arising from the defense
I THE TRIAL COURT ERRED IN MISAPPRECIATING (SIC)
which the offended party might make. In the case at bar, the attack on
THE FACTS OF THE CASE. Magdalino Olos was treacherous, because he was caught off guard and was
therefore unable to defend himself, as testified to by the prosecution
II. THE TRIAL COURT ERRED IN CONCLUDING THAT THERE witnesses and as indicated by the wounds inflicted on him.
WAS CONSPIRACY TO EXTORT RANSOM IN THIS CASE.

III. THE TRIAL COURT ERRED IN CONSIDERING THE


UNCOUNSELLED CONFESSION OF EVANGELINE PADAYHAG. Culpability of Jose Tan
The prosecution was not able to establish conspiracy in the killing of the
IV. THE TRIAL COURT GRAVELY ERRED IN IMPOSING THE
victim; thus, Appellant Jose Tan is guilty only as an accomplice. According
DEATH PENALTY IN THE CASE AT BAR.[7] to the widow Ofelio Olos, she even heard Jose Tan telling and pleading with
his brother to stop his attack and stabbing of the victim. The most therefore
that said accused could be liable for is merely that of an accomplice, who, Issue:
not being a principal cooperated in the execution of the offense by previous
and simultaneous acts, that in this case, by his stoning the victim Modesto Whether or not self defense can be used as a justifying circumstance in this
Olos and hitting him on the neck. However, the accused Jose Tans act of case.
stoning was neither a direct participation nor indispensable to the killing of Whether Jose Tan can be considered as an accomplice in this case.
the victim. Also, as held by the Supreme Court, when doubt exists whether
an accused acted as principal or accomplice, the court should favor the
lesser or milder identity (People vs. Irenea, G.R. No. 44410, August 5,
1988). Case # 47 People v. Abarri

The penalty of Appellant Jose Tan as an accomplice is one degree lower G.R. No. 90185 March 1, 1995
than that of the principal, which in murder cases is reclusion temporal, in its
maximum period, to death. Considering that he is entitled to the privileged
mitigating circumstance of minority, because he was only sixteen years old Art. 8 Conspiracy and proposal to commit a felony
when the crime was committed, the trial court should have lowered his
penalty by two degrees, i.e. prision correccional maximum to prision Art. 17 Principals
mayor medium. Likewise, he is entitled to the benefits of the Indeterminate
Sentence Law.
Since no aggravating or mitigating circumstance was proven, the imposable Doctrine/ Ruling:
penalty on Norlito Tan is reclusion perpetua.
A conspiracy exists when two or more persons come to an agreement
concerning the commission of a felony and decide to commit it. Proof of the
Facts: agreement need not rest on direct evidence, as the agreement itself may be
inferred from the conduct of the parties disclosing a common understanding
among them with respect to the commission of the offense.
On the 6th day of September, 1993, in Barangay Gatbo, Municipality of
Ocampo, Province of Camarines Sur, Philippines, Norlito Tan and Jose Tan
as an accomplice, with intent to kill, with treachery and evident
premeditation, conspiring, confederating together and mutually helping one The common intent of robbing the victim and committing the acts of
another, did then and there, wilfully, unlawfully and feloniously attack, lasciviousness can be inferred from their behaviors.
assault, stone and stab with a deadly weapon one Magdaleno Rudy Olos
alias Modesto Olos, thereby inflicting upon the latter mortal wounds on the
different parts of his body which caused his death, to the damage and
prejudice of the heirs of the offended party in such amount as may be Abarri and Andales each poked a knife at Gan's neck and forcibly brought
proven in court. her to the vacant lot. The other appellants followed them and watched while
Abarri divested the victim of her valuables. After robbing the victim, Abarri
On December 14, 1995, Jose Tan was arrested in Ocampo, Camarines with the use of a "balisong" tore the upper portion of the victim's blouse and
Sur. Subsequently, Norlito Tan was arrested on April 1, 1996. They both all the other appellants participated in removing her clothes, pawing her and
pleaded not guilty. biting her nipples.

The presence or absence of lewd designs is inferred from the nature of the
acts themselves and the environmental circumstances. We find that the acts rest of the blouse and pulling down her pants. The torn blouse was used to
of appellants in striping naked and hogtying the victim and touching her tie her mouth, hands and feet. When she was completely naked, the accused
private parts constitute lewd designs. started touching her private parts.

Abarri opened Gan's handbag and took a bunch of keys, which included the
key for her store at Carmen Planas Street in Binondo, Manila. He also got
However, in the case of Andales, the acts of lasciviousness committed by her watch valued P2,000.00, necklace valued at P5,000.00 and wallet
him culminated in the raping of the victim when he was left alone with her. containing P250.00.
Nothing in the records show that the other accused had knowledge or were
aware of the rape committed by Andales. Consequently, he alone is guilty of
robbery with rape.
After robbing Gan, appellants left except Andales. Before leaving,
Cawaling told Andales: "Nognog, (referring to Andales) bahala ka na,
sampung taon na rin na hindi nakakatikim 'yan, makatas pa 'yan."
Likewise, we do not regard the remarks made by Cawaling to Andales
as sufficient to make him a principal by inducement or a co-
conspirator. Before a remark can produce such an effect, the same must
be of a nature and uttered in such a manner as to become the Andales then dragged Gan to a dark spot and after loosening the tie on her
determining cause of the crime, as to make the utterance a command legs, raped her twice. After satisfying his lust, Andales left.
from a superior to a subordinate. In the case at bench, it appears that
the decision of Andales to rape the victim had been made before
Cawaling uttered the remarks. Cawaling was then leaving the place Gan waited for about 20 minutes before she started to roll over to the
with Abarri, Estrada and Pajalago while Andales purposely stayed middle of the lot. In the process, the tie on her mouth loosened and she was
behind with the victim. There is not even a showing that Cawaling had able to shout for help. Responding to her cries, neighbors came and untied
any moral influence over Andales. her hands.

Facts: Meanwhile, at around 8:30 P.M. of the same day, Barangay Captain Anita
Alejo was informed by a resident that somebody was opening the store of
On October 14, 1988, at around 7:30 P.M., while Gregoria Gan was walking Gan. Repairing at the place, Alejo saw Abarri and Estrada. She noticed that
along 4th Avenue, Kalookan City on her way home, Ernesto Abarri and the door of the store had been partly opened. When she asked the two what
Ronnie Andales stopped her and each poked a knife at her neck. Abarri then they were doing there, Abarri answered that Gan instructed them to get the
grabbed Gan's bag and warned: "Kung gusto mong mabuhay, huwag kang latter's pants. Alejo brought them to the barangay hall for investigation.
sisigaw." Gan was dragged by the two and brought inside a fenced, vacant Upon further questioning, Abarri admitted to forcibly bringing Gan to a
lot strewn with garbage and covered with tall grass. Clemente Cawaling, vacant lot and binding her arms and legs. Alejo turned over the two to the
Conrado Estrada and Joselito Pajalago were former employees of Gan. custody of the police detachment in Binondo.
Once inside the vacant lot, Abarri, with the use of a "balisong, " tore the
upper portion of Gan's blouse. The other accused then started tearing the
The police brought Abarri and Andales to the crime scene. However, Gan
was no longer there when they arrived. The police proceeded to Gan's house Issue:
where the latter positively identified the two as among those persons who Whether or not under Article 48 of the Revised Penal Code rebellion may
robbed her. properly be complexed with a common offense, murder.

Issue:
#49 Santiago v Garchitorena
Are the accused guilty of robbery with rape?
ART 10 RPC - Supplementing Special Laws

CONTINUING CRIME - For delito continuado to exist there should be a


Case No. 48 plurality of acts performed during a period of time; unity of penal provision
Enrile vs. Salazar violated; and unity of criminal intent or purpose, which means that two or
186 SCRA 217 more violations of the same penal provisions are united in one and same
G.R. No. 92163 64 instant or resolution leading to the perpetration of the same criminal
June 5, 1990 purpose or aim

Intro: a delito continuado consists of several crimes but in reality there is only one
Article 48 of our Penal Code cannot be applied in the case at bar. If murder crime in the mind of the perpetrator.
were not complexed with rebellion, and the two crimes were punished
separately (assuming that this could be done), the following penalties would FACTS:
be imposable upon the movant, namely: (1) for the crime of rebellion, a fine
not exceeding P20,000 and prision mayor, in the corresponding period, Petitioner was charged with performing a single criminal act that of her
depending upon the modifying circumstances present, but never exceeding approving the application for legalization of 32 aliens not qualified under
12 years of prision mayor, and (2) for the crime of murder, reclusion the law to enjoy such privilege under EO 324, causing undue injury to one
temporal in its maximum period to death, depending upon the modifying offended party, the Government, done on a single day, i.e., on or about
circumstances present. in other words, in the absence of aggravating October 17, 1988.
circumstances, the extreme penalty could not be imposed upon him.
However, under Article 48 said penalty would have to be meted out to him, According to petitioner, unless she was furnished with the names and
even in the absence of a single aggravating circumstance. Thus, said identities of the aliens, she could not properly plead and prepare for trial.
provision, if construed in conformity with the theory of the prosecution,
would be unfavorable to the movant. The information was then amended reproducing verbatim the allegation of
the original information, except that instead of the word "aliens" in the
Facts: original information each amended information states the name of the
A warrant was issued charging Senator Enrile, the spouses Rebecco and individual whose stay was legalized.
Erlinda Panlilio, and Gregorio Honasan with the crime of rebellion with
murder and multiple frustrated murder allegedly committed during the At the hearing of the motion for a bill of particulars, the public prosecutors
period of the failed coup attempt from November 29 to December 10, 1990. manifested that they would file only one amended information embodying
The petitioner's counsel contended that the petitioner is charged with a the legalization of stay of the 32 aliens. As stated in the Order dated
crime that does not exist in the statute books, therefore, the same must be November 12, 1992 of the Sandiganbayan (First Division):
dismissed as it is just a mere flight of rhetoric.
DOCTRINE: Absolute pardon not only blots out the crime committed but
On the matter of the Bill of Particulars, the prosecution has conceded removes all disabilities resulting from the conviction.
categorically that the accusation against Miriam Defensor Santiago consists
of one violation of the law represented by the approval of the applications FACTS: The CFI of Rizal found respondent Santos guilty of estafa,
of 32 foreign nationals for availment (sic) of the Alien Legalization however, he continued to be a registered elector in the city of Malabon,
Program. In this respect, and responding directly to the concerns of the Rizal and was seated as the municipal president from 1934 to 1937. On
accused through counsel, the prosecution is categorical that there will not be 1938, Commonwealth Act No 357 or the Election Code was approved
32 accusations but only one . . . ( which disqualifies the respondent from voting for having been declared by
final judgment guilty of any crime against property. The
ISSUE: respondent applied to the President for an absolute pardon and granted the
Is the consolidation of cases proper for violation of EO 324? / Was there petition restoring the respondent to his full civil and political rights, except
only one offense committed? that with respect to the right to hold public office or employment, he will be
eligible for appointment only to positions which are clerical or manual in
RULING: nature and involving no money or property responsibility.
YES. Under Article 10 of the Revised Penal Code, the Code shall be
supplementary to special laws, unless the latter provide the contrary. Hence, On 1940, Cristobal filed a petition for the exclusion of the name of Santos
legal principles developed from the Penal Code may be applied in a from the list of voters in precinct no. 11 of Malabon, Rizal on the ground
supplementary capacity to crimes punished under special laws. that the latter is disqualified. After hearing, the court denied the petition for
exclusion and declared that the pardon extended in favor of respondent has
Applying the concept of delito continuado had the effect of excluding him from the disqualification created by the
Election Code.
The 32 Amended Informations aver that the offenses were committed on the
same period of time, i.e., on or about October 17, 1988. The strong ISSUE: Whether or not the pardon not only blots out the
probability even exists that the approval of the application or the crime committed but removes all disabilities resulting from the conviction?
legalization of the stay of the 32 aliens was done by a single stroke of the
pen. Likewise, the public prosecutors manifested at the hearing the motion
for a bill of particulars that the Government suffered a single harm or injury.
54. RICARDO PARULAN, petitioner, vs. DIRECTOR OF PRISONS,
Criminal Case No. 16698 is MODIFIED in the sense that the Office of the respondent. G.R. No. L-28519 February 17, 1968
Special Prosecutor of the Office of the Ombudsman is directed to Ruling: There are crimes which are called transitory or continuing offenses
consolidate the 32 Amended Informations into one information charging
because some acts material and essential to the crime occur in one province
only one offense under the original case number,
Case # 52 and some in another, in which case, the rule is settled that the court of either
province where any of the essential ingredients of the crime took place has
Cristobal v. Labrador jurisdiction to try the case. There are, also, crimes which although all the
elements thereof for its consummation may have occurred in a single place,
G.R. No. L-47941 December 7, 1940 yet by reason of the very nature of the offense committed, the violation of
the law is deemed to be continuing. The crime of evasion of service of
sentence is an example of the latter kind of crime - when the prisoner in his
ABSOLUTE PARDON attempt to evade the service of the sentence imposed upon him by the courts
and thus defeat the purpose of the law, moves from one place to another; necessarily prove each of the component offenses with the same precision
for, in this case, the act of the escaped prisoner is a continuous or series of that would be necessary if they were made the subject of separate
acts, set on foot by a single impulse and operated by an unintermittent force, complaints.
however long it may be. It may not be validly said that after the convict
shall have escaped from the place of his confinement the crime is fully Mitigating Circumstance of voluntary plea of guilty in single and
consummated, for, as long as he continues to evade the service of his indivisible penalty. The presence of mitigating circumstance of voluntary
sentence, he is deemed to continue committing the crime, and may be plea of guilty has no effect or disregarded incase the penalty imposable is
arrested without warrant, at any place where he may be found. Rule 113 of single and indivisible.
the Revised Rules of Court may be invoked in support of this conclusion, Automatic Review in case the penalty imposed is death, reclusion
for, under section 6[c] thereof, one of the instances when a person may be perpetua or life imprisonment. When the penalty of death, reclusion
arrested without warrant is where he has escaped from confinement. perpetua or life imprisonment is imposed, the case is automatically
Undoubtedly, this right of arrest without a warrant is founded on the forwarded to the Supreme Court on automatic review. However, such
principle that at the time of the arrest, the escapee is in the continuous act of automatic review to the Supreme Court does not bar the referring of the
committing a crime evading the service of his sentence. case to the Court of Appeals for intermediate disposition.
ISSUE: Whether or not the Court of First Instance of Manila has Searching inquiry incase of plea of guilty in a crime having a penalty of
jurisdiction to hear and decide the case against Parulan? death, reclusion perpetua or life imprisonment. There is thus no hard and
FACTS: Petitioner Parulan was confined in the state penitentiary at fast rule as to how a judge may conduct a "searching inquiry." As long as
Muntinlupa, Rizal, serving a sentence of life imprisonment which, however, the voluntary intent of the accused and his full comprehension of the
was commuted to 20 years by the President. He was then transferred to Fort consequences of his plea are ascertained.
Bonifacio in Makati. In October 1964, while still serving his sentence, he
escaped. He was eventually recaptured in Manila. Consequently, he was
prosecuted for the crime of evasion of service of sentence and was found
Facts: In January 14, 2004, while AAA was on her way to school, appellant
guilty. As a recourse, he filed a petition for the issuance of writ of habeas
kidnapped AAA by deceiving the latter that they would would go to
corpus directed against respondent Director of Prisons.
Jollibee, but in fact appellant brought AAA to a house in Imus, Cavite. AAA
PEOPLE OF THE PHILIPPINES vs. RENATO TALUSAN y was thereafter under appellants control and custody for eight days during
PANGANIBAN which he abused her by inserting his finger inside her vagina on a daily
basis before breakfast, despite her resistance.
G.R. No. 179187 July 14, 2009
AAA having failed to return home, her stepfather BBB went to search for
CARPIO MORALES, J.: the former. A neighbor then informed him that he saw appellant with AAA
while the latter was on her way to school. At dawn of the following day,
January 23, 2004, appellant, who was with AAA, was apprehended.

Doctrine(s): Special complex crime. Where the law provides a single An information for kidnapping with rape was filed against appellant by on
penalty for two or more component offenses, the resulting crime is called a the basis of the report submitted by the medico legal.
special complex crime. In a special complex crime, the prosecution must
Upon arraignment, appellant, with the assistance of his counsel de oficio, 2 A special law is defined as a penal law which punishes acts not
entered a plea of guilty. The lower court thereupon conducted a searching defined and penalized by the Revised Penal Code. In the present
inquiry into the voluntariness of appellants plea, and despite repeated case, petitioner was charged with and convicted of Conduct
questions and just as repeated answers showing that appellant understood Unbecoming an Officer and Gentleman (96th Article of War) and
his plea and its consequences, the trial court still ordered the prosecution to, Violation of the 97th Article of War, or Conduct Prejudicial to
as it did, present evidence. Good Order and Military Discipline, both of which are not defined
and penalized under the Revised Penal Code. The corresponding
Issue: Whether or not a searching inquiry is required in case of plea of penalty imposed by the General Court Martial, which is two (2)
guilty where the penalty imposed is death, reclusion perpetua or life years of confinement at hard labor is penal in nature. Therefore,
imprisonment? absent any provision as to the application of a criminal concept in
the implementation and execution of the General Court Martial's
decision, the provisions of the Revised Penal Code, specifically
Article 29 should be applied.

3 Article 29 of the Revised Penal Code in the Articles of War is in


accordance with the Equal Protection Clause of the 1987
G.R. No. 198554 July 30, 2012 Constitution. According to a long line of decisions, equal
protection simply requires that all persons or things similarly
MAJOR GENERAL CARLOS F. GARCIA, AFP (RET.), Petitioner, situated should be treated alike, both as to rights conferred and
vs. responsibilities imposed. It requires public bodies and institutions
THE EXECUTIVE SECRETARY, to treat similarly situated individuals in a similar manner. The
purpose of the equal protection clause is to secure every person
within a states jurisdiction against intentional and arbitrary
Doctrines: discrimination, whether occasioned by the express terms of a
statute or by its improper execution through the states duly-
1 A court-martial case is a criminal case and the General Court constituted authorities
Martial is a court akin to any other courts. The General Court
Martial is a court within the strictest sense of the word and acts as
a criminal court. On that premise, certain provisions of the Revised Facts:
Penal Code, insofar as those that are not provided in the Articles of Major General Carlos Garcia of the AFP was charged in a Court Martial for
War and the Manual for Courts-Martial, can be supplementary. failure to disclose all his existing assets in his Sworn Statement of Assets
Under Article 10 of the Revised Penal Code: Art. 10. Offenses not and Liabilities and Net Worth for the years 2002 and 2003. After six (6)
subject to the provisions of this Code.Offenses which are or in years and two (2) months of preventive confinement, Garcia was found
the future may be punishable under special laws are not subject to guilty of the charged offenses and was dishonorably discharged from the
the provisions of this Code. This Code shall be supplementary to service, forfeited all pay and allowances due and confined at hard labor at
such laws, unless the latter should specially provide the contrary. such place the reviewing authority may direct for a period of two (2) years.
Issue: WON the period of preventive confinement of Garcia shall be The settled rule is that when an accused pleads to the charge, he is
credited to the sentence imposed by the court martial. deemed to have waived the right to preliminary investigation
and the right to question any irregularity that surrounds it.
Ruling: Yes. Applying the provisions of Article 29 of the Revised Penal
Code, the time within which the petitioner was under preventive Under Article 310 of the Revised Penal Code, the penalty for
confinement should be credited to the sentence confirmed by the Office of
Qualified Theft is two degrees higher than that specified in
the President. The period of confinement of six years shall be credited in his
Article 309.
favor and deducted from the two (2) years to which the accused was
sentenced.

FACTS:

Ferdinand Cruz was a Marketing Manager of Porta-Phone Rentals, Inc., a


corporation engaged in the lease of cellular phones. He went to his office,
obtained a pad of official receipts from the collection officers table. He
delivered the communication equipment and received the PhP15,000.00
payment and issued a receipt even though he not authorized to receive cash
payments and issued receipts. He then failed to deliver the cash to his
office. When he was confronted, Ferdinand admitted that he deposited the
FERDINAND A. CRUZ, Petitioner, v. THE PEOPLE OF THE
amount to his personal bank account. He was instructed to remit the amount
PHILIPPINES, Respondnet.
which he failed to do claiming that the company has paid his
G.R. No. 176504, Spetember 03, 2008 reimbursements.. The company sent a demand letter which he answered
stating that he already remitted the amount to the accounting supervisor
which the latter denied. He refused to turn over the amount despite demands
made by the officers of the company which prompted the company to file
ARTICLE
the a case of Estafa/Falsification of Documents. According to the
Article 310 of the Revised Penal Code, theft is qualified when it is, Information filed before the RTC of Makati, Ferdinand Cruz as a Marketing
among others, committed with grave abuse of confidence, to wit: Manager of Porta-Phone Rentals, Inc. had access to the funds of the
corporation and with grave abuse of confidence, steal and carry away the
amount of PhP15,000.00. He entered a plea of not guilty during
arraignment, and the witnesses for the prosecution testified that he received
ART. 310. Qualified theft. - The crime of theft shall be
the said amount from Hemisphere-Leo Burnett and issued a receipt but
punished by the penalties next higher by two degrees than
failed to turn over the amount to Porta-Phone Rentals. The prosecution
those respectively specified in the next preceding article,
changed the case to Qualified Theft.
if committed x x x with grave abuse of confidence x x x.
Ferdinand Cruz in his defense alleged that he issued an acknowledgment
receipt to Hemisphere and remitted the amount to the accounting supervisor.
He was asked by the supervisor to sign the official receipt because he was
the one who closed the deal.
The RTC found Ferdinand guilty beyond reasonable doubt for the crime of proper to charge Ferdinand with qualified theft. Since the same
QUALIFIED THEFT. He filed a Motion for New Trial on the grounds of allegations and evidence were proffered by the complainant in the
(1) absence of a preliminary investigation for the crime of qualified theft qualified theft, there is no need for Ferdinand to be given the
and (2) newly discovered evidence the testimony of a former employee of opportunity to submit counter-affidavits anew, as he had already
Hemisphere testifying on Ferdinands signing of an acknowledgment answered said allegations when he submitted counter-affidavits for
receipt. The RTC revived and reinstated the conviction of Ferdinand. Upon the original indictment of estafa/falsification of private documents.
appeal, the Court of Appeals affirmed the RTC Decision.

2 The elements of the crime of theft are the following: (1) there was
ISSUE: a taking of personal property; (2) the property belongs to another;
(3) the taking was without the consent of the owner; (4) the taking
1 Whether Ferdinand was denied of due process when he was was done with intent to gain; and (5) the taking was accomplished
indicted for qualified theft even as he was initially investigated for without violence or intimidation against the person or force upon
estafa/falsification of private documents. things.12 Under Article 310 of the Revised Penal Code, theft is
2 Whether Ferdinands guilt was not established beyond reasonable qualified when it is, among others, committed with grave abuse of
doubt. confidence, to wit:
3 Whether the Indeterminate Sentence Law should be applied.

ART. 310. Qualified theft. - The crime of theft shall be punished by


the penalties next higher by two degrees than those respectively
HELD: specified in the next preceding article, if committed x x x with
grave abuse of confidence x x x.
1 Ferdinand was not denied of due process. The settled rule is that
when an accused pleads to the charge, he is deemed to have
waived the right to preliminary investigation and the right to
The prosecution established, beyond the shadow of doubt that
question any irregularity that surrounds it. In the instant case,
Ferdinand took and kept the fifteen thousand peso-collection from
Ferdinand did not present evidence that arraignment was forced
the companys client.
upon him. On the contrary, he voluntarily pleaded to the charge
and actively participated in the trial of the case

3 The RTC imposed on petitioner the indeterminate penalty of 10


Years and 1 Day of prision mayor as minimum to 14 Years, 8
It is not correct for Ferdinand to claim that preliminary
Months and 1 Day of reclusion temporal, as maximum.
investigation on the charge of qualified theft was not accorded
him. The truth is, Ferdinand was able to answer the initial charge Under Article 310 of the Revised Penal Code, the penalty for
of estafa/falsification of private documents through his counter- Qualified Theft is two degrees higher than that specified in Article
affidavits. Based on the same complaint affidavit and the same sets 309. Paragraph 1 of Article 309 provides that if the value of the
of evidence presented by the complainant, the prosecutor deemed it thing stolen is more than P12,000.00 but does not exceed
P22,000.00, the penalty shall be prision mayor in its minimum and knew. There being no actual detention or confinement, the appellant may be
medium periods. In this case, the amount stolen was P15,000.00. convicted only of grave coercion.
Two degrees higher than prision mayor minimum and medium is
reclusion temporal in its medium and maximum periods. Applying
the Indeterminate Sentence Law, the minimum shall be prision
mayor in its maximum period to reclusion temporal in its
minimum period or within the range of 10 years and 1 day to 14
years and 8 months. There being neither aggravating nor mitigating
circumstance in the commission of the offense, the maximum No Motive to "Kidnap"
period of the indeterminate sentence shall be within the range of 16
years, 5 months and 11 days to 18 years, 2 months and 20 days.
The minimum penalty imposed by the RTC is correct. However,
the maximum period imposed by RTC should be increased to 16 Motive is not an element of the crime. Furthermore, motive becomes
years, 5 months and 11 days. material only when the evidence is circumstantial or inconclusive, and there
is some doubt on whether a crime has been committed or whether the
accused has committed it. Indeed, motive is totally irrelevant when ample
direct evidence sustains the culpability of the accused beyond reasonable
doubt. In this case, the identity of appellant is not in question. He himself
admitted having taken Yvonne to Maco Central Elementary School.
PEOPLE OF THE PHILIPPINES v. ARNULFO ASTORGA

G.R. No. 110097, 22 December 1997

Kidnapping or Coercion?

Introduction (Provisions involved, principles, jurisprudence):


The prosecution failed to prove one essential element of kidnapping the
fact of detention or the deprivation of liberty. Under Article 267 of the
Revised Penal Code, the elements of kidnapping are as follows:
Actual detention or "locking up" is the primary element of kidnapping. If
the evidence does not adequately prove this element, the accused cannot be
held liable for kidnapping. In the present case, the prosecution merely
proved that appellant forcibly dragged the victim toward a place only he 1 That the offender is a private individual.
2 That he kidnaps or detains another, or in any other A review of the events as narrated by the prosecution witnesses ineluctably
manner deprives the latter of his liberty. shows the absence of "locking up." It is clear that the appellant and the
victim were constantly on the move. There is no actual confinement or
3 That the act of detention or kidnapping must be illegal. restraint of the victim, which is the primary element of
kidnapping. Appellant's apparent intention was to take Yvonne against her
4 That in the commission of the offense, any of the following will towards the direction of Tagum. The evidence does not show that
circumstances is present: appellant wanted to detain Yvonne; much less, that he actually detained her.
Appellant's forcible dragging of Yvonne to a place only he knew cannot be
1 That the kidnapping or detention lasts for more than said to be an actual confinement or restriction on the person of Yvonne.
five (5) days; or There was no "lockup." Accordingly, appellant cannot be convicted of
kidnapping under Article 267 of the Revised Penal Code.
2 That it committed simulating public authority; or

3 That any serious physical injuries are inflicted upon


the person kidnapped or detained or threats to Rather, the felony committed in this case is grave coercion under Article
kill him are made; or 286 of the same code. Grave coercion or coaccion grave has three elements:
(a) that any person is prevented by another from doing something not
4 That the person kidnapped or detained is a minor, prohibited by law, or compelled to do something against his or her will, be it
female, or a public officer. right or wrong; (b) that the prevention or compulsion is effected by
violence, either by material force or such a display of it as would produce
intimidation and, consequently, control over the will of the offended party;
and (c) that the person who restrains the will and liberty of another has no
The Spanish version of Article 267 of the Revised Penal Code uses the right to do so or, in other words, that the restraint is not made under
terms "lockup" (encerrar) rather than "kidnap" (secuestrar or raptar). authority of a law or in the exercise of any lawful right. When appellant
Lockup is included in the broader term of "detention," which refers not only forcibly dragged and slapped Yvonne, he took away her right to go home to
to the placing of a person in an enclosure which he cannot leave, but also to Binuangan. Appellant presented no justification for preventing Yvonne from
any other deprivation of liberty which does not necessarily involve locking going home.
up. Likewise, the Revised Penal Code was originally approved and enacted
in Spanish. Consequently, the Spanish text is controlling in cases of conflict
with the English version, as provided in Section 15 of the Revised
Administrative Code.

Brief facts:
While there was a brown out, accused or "Boy" Astorga told Yvonne Traya,
who was only eight (8) years old at that time and who stays with her
grandparents and so with her parents at Sitio Binuangan, Maco, to go with Brief issues:
him to buy candy and immediately grabbed and held Yvonnes hand as the
latter did not answer, placed his hand on her shoulder, covered her mouth,
and went and walked inside the compound of Maco Elementary School, and
later, there being no person around the gate, accused brought her out to the Whether the accuseds motive to kidnap Yvonne Traya is relevant to convict
highway and walked towards the direction of Tagum which is the opposite him for the crime of kidnapping. No, motive is not relevant.
direction towards her grandparent's house at Binuangan, hence Yvonne cried
and protested that she must go home but the accused did not heed her plea.

Whether Yvonne Traya was not detained, locked-up or deprived of her


liberty so as not to convict the accused for the crime of kidnapping but only
While the accused and Yvonne were walking in the situation as described, for grave coercion. Yes!
somewhere near the Luponlupon bridge, they met some group of
youngsters-men and the said group was suspicious about the man who was
bringing a child, thus the said group decided to follow them. Accused
hurriedly walked fast with Yvonne, and to prevent from being overtaken, he
carried the victim and ran but they were chased and were overtaken.
ORTEGA V. PEOPLE (CRIMINAL)

ARTICLE 12 - EXEMPTING CIRCUMSTANCES


Edwin Fabila declared that Jonathan, one of his companions with others in
chasing, asked the accused where they were bound and he answered Section 6 of RA 9344 clearly and explicitly provides:
towards Binuangan but the group noticed something suspicious because
Section 6. Minimum Age of Criminal Responsibility - A child 15 years of
their destination was already towards Tagum which is an opposite direction age or under at the time of the commission of the offense shall be exempt
to Binuangan. from criminal liability. However, the child shall be subjected to an
intervention program pursuant to Section 20 of this Act.

A child above 15 years but below 18 years of age shall likewise be exempt
from criminal liability and be subjected to an intervention program, unless
Jonathan, one of those who chased, knew Yvonnes family and he got from
he/she acted with discernment, in which case, such child shall be subjected
the accused Yvonne who showed some resistance, and the group brought the to the appropriate proceedings in accordance with this Act
accused and Yvonne to Yvonnes home at Binuangan.

ORTEGA vs PEOPLEGR No. 151085August 20, 2008


PEOPLE OF THE PHILIPPINES vs. IRVIN TADULAN y EPAN
FACTS: G.R. No. 117407 | April 15, 1997
The petitioner, Joemar Ortega, who was then 14 years old, was charged with the crime of PADILLA, J.
rape
for allegedly raping AAA, who was about 8 years old. That the rape happened in 3
occasions. The RTC ruled that the petitioner is guilty beyond reasonable
doubt in the crime of rape and is sentenced to reclusion temporal. The CA affirmed DOCTRINE / RULING:
the ruling of the trial court. During the pendency of the case in the SC, RA 9344 Juvenile Alibi; It has been held time and again that for alibi to prosper as a defense
Justice and Welfare Act was enacted that establishes a comprehensive system to manage the accused must show that he was so far away that he could not have been
children in conflict with the law. At the case at bar, because the petitioner was a minor physically present at the place of the crime, or its immediate vicinity at the
under 15 years of age at the commission of the crime, he can be relieved from criminal time of its commission (People vs. Tasurra, 192 SCRA 266). In this case,
liability. however, it is not so situated, for according to him he was at the plant of the
Republic Asahi Glass Corporation in Barangay Pinagbuhatan, Pasig,
ISSUE: which is but a few kilometers from Barangay Sumilang of the same
Whether or not the petitioner can avail exempting circumstance provided by the newly municipality where the crime was committed.
enacted law on minors in conflict with law.
HELD Rape; Accused-appellant tries to discredit the victims testimony by
: Yes questioning her behavior after she was allegedly raped. The court ruled that
RATIO it is not proper to judge the actions of children who have undergone
: The petitioner can avail the exempting circumstance that will relieve him from criminal traumatic experience by the norms of behavior expected under the
liability because the law enacted was favorable to the accused, and is therefore retroactive circumstances from mature people. The range of emotion shown by rape
in application. Juvenile Justice and Welfare Act provides that a child under 15 years of victims is yet to be captured even by the calculus. It is thus unrealistic to
age in the commission of the offense shall be exempt from criminal liability, but is expect uniform reactions from rape victims.It should be borne in mind, in
subject to an intervention program. Exemption from criminal liability, however, does not this connection, that the victim was only a naive nine (9) year old child
include exemption from civil liability. Section 64 of the newly enacted law also provides when the crime was committed on her. She considered the accused as a
that cases of children under 15 years of age at the commission of the crime, shall friend, almost like a relative, as in fact she called him Tito Loloy.
immediately be dismissed and the child shall be referred to the appropriate
local social welfare and development officer. The Court therefore held that As correctly observed by the Solicitor General: (A)s regards the acts
the case against Joemar Ortega is hereby DISMISSED. Petitioner is hereby imputed to Estela, the delay of seven (7) days from the date of her
referred to the local social welfare and development officer of the locality for the knowledge of the rape incident on 4 April 1992 in reporting to the
appropriate intervention program authorities the rape of her daughter is excusable. At that time, she was not
yet certain of the steps she would take considering the delicate nature of the
problem they were facing (citing People v. Danguilan, 218 SCRA 98;
People v. Joaquin, Jr., 225 SCRA 179). Besides, we have ruled that a delay
in prosecuting the rape is not indicative of fabricated charges.

Pardon; It is clear to the mind of this Court that the complainant has not
expressly pardoned the said accused. Besides, there are authorities holding
Case No. 65 that pardon must be granted not only by the parents of an offended minor
but also by the minor herself in order to be effective as an express pardon
under Art. 344 of the Revised Penal Code. Thus, in the case of People vs. Whether or not the court erred in disregarding the defense of pardon and
Lacson, Jr., (C.A.) 55 O.G. 9460, we find the following words: Neither alibi of the accused?
must we be understood as supporting the view that the parents alone can PANAGUITON, JR. v. DOJ
extend a valid pardon. Far from it, for we, too are of the belief that the
pardon by the parents, standing alone, is inefficacious. It was also held in Facts:
another case, that The express pardon of a person guilty of attempted
In 1992, Rodrigo Cawili borrowed various sums of money from petitioner
abduction of a minor, granted by the latters parents, is not sufficient to
remove criminal responsibility, but must be accompanied by the express Panaguiton, Jr. On January 8, 1993, Cawili and Ramon C. Tongson
pardon of the girl herself. (U.S. vs. Luna, 1 Phil. 360). (Tongson) jointly issued in favor of Panaguiton, Jr. three (3) checks in
payment of the said loans. Upon presentment for payment on March 18,
1993, the checks were dishonored.
FACTS:
Complainant Estela Santos owns a house at No. 6 Dr. Garcia St., in On August 24, 1995, Panaguiton, Jr. filed a complaint against Cawili and
Barangay Sumilang, Pasig, she resides with her common-law husband and Tongson for violating Batas Pambansa (B.P.) Bilang 22 before the Quezon
their minor daughter, Maristel Cruz. Behind the said house, complainant City Prosecutor's Office. However, on March 15, 1999, Assistant City
also owns a three-door apartment building, one unit of which was rented Prosecutor Sampaga held that the case had already prescribed pursuant to
and occupied by accused Irvin Tadulan, his wife Adefa Tadulan and their Act No. 3326, as amended, which provides that violations penalized by B.P.
three children name Dianne, Angie and Bochoy who were aged 10, 9 and 5, Blg. 22 shall prescribe after four (4) years starting on the date the checks
respectively. In 1992 complainants daughter, Maristel Cruz was about nine were dishonored (March 18, 1993) Panaguiton, Jr. appealed to the DOJ but
(9) year old and was in grade school. She often played with the accuseds it dismissed the case on the same grounds of prescription.
children in the vicinity of their house and the apartment building.
The DOJ explained that Act No. 3326 applies to violations of special acts
That on or about the 2nd day of April, 1992 in the Municipality of Pasig, that do not provide for a prescriptive period for the offenses thereunder.
Metro Manila, Philippines and within the jurisdiction of this Honorable Since B.P. Blg. 22, as a special act, does not provide for the prescription of
Court, armed with a knife, with lewd design and by means of force, threats the offense it defines and punishes, Act No. 3326 applies to it, and not Art.
and intimidation, did then and there willfully, unlawfully and feloniously 90 of the Revised Penal Code which governs the prescription of offenses
have sexual intercourse with one Maristel Cruz, a minor, nine (9) years old, penalized thereunder. The DOJ also cited the case of Zaldivia v. Reyes, Jr.,
without her consent and against her will. wherein the SC ruled that the proceedings referred to in Act No. 3326, as
amended, are judicial proceedings, and not the one before the prosecutor's
Estela Santos immediately informed the wife of Irvin Tadulan that her office (administrative proceedings). Therefore, the filing of a complaint
husband has raped her daughter. She further informed Adefa Tadulan that
with the Office of the City Prosecutor of Quezon City does not interrupt the
she would not take action against the latters husband if they would vacate
running of the prescriptive period for violation of B.P. Blg. 22. Hence, this
the apartment unit right away. Adefa Tadulan later on met with Estela
Santos and told her that she had driven away Irvin Tadulan, but requested Petition for Review.
that she and her children be allowed to stay until Saturday, April 11, 1992.
Estela Santos thereafter noted, however, that Irvin Tadulan was still coming
home to the apartment unit every night despite the promise of his wife that Issues:
she herself would call the police should he ever come back to the place.
Whether or not the offense of violation of B.P. Blg. 22 has already
ISSUE: prescribed per Act No. 3326
The prescription shall be interrupted when proceedings are instituted against
the guilty person, and shall begin to run again if the proceedings are
Ruling: dismissed for reasons not constituting jeopardy.
NO.
In cases involving violations of R.A. No. 3019 committed prior to the
As correctly stated by the DOJ, Act No. 3326 is the law applicable to February 1986 Edsa Revolution that ousted President Ferdinand E. Marcos,
offenses under special laws which do not provide for their own prescriptive we ruled that the government as the aggrieved party could not have known
periods. However, as ruled in the cases of Ingco v. Sandiganbayan, Sanrio of the violations at the time the questioned transactions were made (PCGG
Company Limited v. Lim, and Securities and Exchange Commission v. vs. Desierto, G.R. No. 140232, January 19, 2001, 349 SCRA 767; Domingo
Interport Resources Corporation, et al, the Court ruled that the v. Sandiganbayan, supra, Note 14; Presidential Ad Hoc Fact Finding
prescriptive period is interrupted by the institution of proceedings for Committee on Behest Loans v. Desierto, supra, Note 16). Moreover, no
preliminary investigation against the accused. Although Panaguiton, Jr. person would have dared to question the legality of those transactions.
went through the proper channels, within the prescribed periods, an Thus, the counting of the prescriptive period commenced from the date of
aggregate period of nine (9) years had elapsed from the time he filed his discovery of the offense in 1992 after an exhaustive investigation by the
complaint-affidavit up to the time the DOJ issued the assailed resolution. Presidential Ad Hoc Committee on Behest Loans.-Presidential Ad Hoc
Aggrieved parties, especially those who do not sleep on their rights and Fact-Finding Committee on Behest Loans v. Desierto (2001)
actively pursue their causes, should not be allowed to suffer unnecessarily
further simply because of circumstances beyond their control.
Notwithstanding whether or not the proceeding is a judicial proceeding or RULING: Respondent Ombudsman committed grave abuse of discretion in
an administrative proceeding, Panaguiton, Jr.s filing of the complaint dismissing the subject complaint on the ground of prescription.
interrupted the running of the prescriptive period.
Respondents members of the PNB Board of Directors and Officers of
NOCOSII are charged with violation of R.A. No. 3019, a special law.
PRESIDENTIAL COMMISSION ON GOOD GOVERNMENT Amending said law, Section 4, Batas Pambansa Blg. 195, increased the
(PCGG) vs. HON. ANIANO A. DESIERTO, ET AL. prescriptive period from ten to fifteen years.

Prescriptive period for Anti-Graft and Corrupt Practices Act (RA No. The issue of prescription has long been laid to rest in the
3019, as amended) aforementioned Presidential Ad Hoc Fact-Finding Committee on Behest
Loans v. Desierto, where the Court held that it was well-nigh impossible for
The applicable law in the computation of the prescriptive period is Section the State, the aggrieved party, to have known the violations of R.A. No.
2 of Act No. 3326, as amended (Act Establishing Prescriptive Periods for 3019 at the time the questioned transactions were made because, as alleged,
Violations of Special Laws and Municipal Ordinances), which provides that the public officials concerned connived or conspired with the "beneficiaries
prescription shall begin to run from the day of the commission of the of the loans. Thus, we agree with the COMMITTEE that the prescriptive
violation of the law, and if the same not be known at the time, from the period for the offenses with which respondents in OMB-0-96-0968 were
discovery thereof and the institution of judicial proceedings for its charged should be computed from the discovery of the commission thereof
investigation and punishment. overwhelm and not from the day of such commission.
The assertion by the Ombudsman that the phrase 'if the same not be known' The respondents failed to submit any responsive pleading before the
in Section 2 of Act No. 3326 does not mean 'lack of knowledge' but that the Ombudsman, prompting Graft Investigator Officer (GIO) I Melinda S.
crime 'is not reasonably knowable' is unacceptable, as it provides an Diaz-Salcedo to resolve the case based on the available evidence. In a
interpretation that defeats or negates the intent of the law, which is written Resolution, GIO Diaz-Salcedo recommended the dismissal of the case on
in a clear and unambiguous language and thus provides no room for the ground of insufficiency of evidence or lack of probable cause against the
interpretation but only application. respondents and for prescription of the offense. Ombudsman Desierto
approved the recommendation. Petitioner filed a Motion for
As to when the period of prescription was interrupted, the second paragraph Reconsideration but it was denied by GIO Diaz-Salcedo, which was
of Section 2, Act No. 3326, as amended, provides that prescription is approved by Ombudsman Desierto.
interrupted 'when proceedings are instituted against the guilty person.
ISSUE: Whether or not the Ombudsman committed grave abuse of
Records show that the act complained of was discovered in 1992. The discretion in ruling that the offense leveled against respondents has
complaint was filed with the Office of the Ombudsman on April 5, 1995, or prescribed.
within three (3) years from the time of discovery. Thus, the filing of the
complaint was well within the prescriptive period of 15 years.

SOCIAL SECURITY SYSTEM G.R. No. 158131


FACTS: On October 8, 1992, President Fidel V. Ramos issued
Administrative Order No. 13 creating the Presidential Ad Hoc Fact-Finding vs
Committee on Behest Loans (Committee) which was tasked to inventory all
behest loans, determine the parties involved and recommend whatever DEPARTMENT OF JUSTICE
appropriate actions to be pursued thereby and Memorandum Order No. 61
August 8, 2007
expanded the functions of the Committee to include the inventory and
review of all non-performing loans, whether behest or non-behest.

Among the accounts referred to the Committee's Technical Working Group RULING
(TWG) were the loan transactions between Northern Cotabato Sugar The Supreme Court ruled that SENCORs criminal liability was not
Industries, Inc. (NOCOSII) and Philippine National Bank (PNB) and the extinguished. The SC argued that for novation to apply there must be an
Committee classified the loans obtained by NOCOSII from PNB as behest original contract to speak of. In this case, the novation does not apply
because of NOCOSII's insufficient capital and inadequate collaterals after it because there was no original contract that can be replaced by a new
had examined and studied all the documents relative to the said loan contract changing the object or principal condition of the original contract,
transactions. substituting the person of the debtor, or subrogating a third person in the
rights of the creditor.
Based on the Sworn Statement of PCGG consultant Orlando Salvador,
petitioner filed with the Office of the Ombudsman the criminal complaint
against respondents on April 5, 1995. Petitioner alleges that respondents
The Court observed that although novation is not one of the means
violated the following provisions of Section 3 (e) and (g) of R.A. No. 3019.
recognized by the Revised Penal Code to extinguish criminal liability, it
may prevent the rise of criminal liability or to cast doubt on the true nature complaint against Martels and their five co-accused for SENCORs non-
of the original basic transaction, provided the novation takes place before payment of contributions. To pay this amount, respondent Martels offered to
the filing of the Information with the trial court. In the case People v. Nery, assign to petitioner a parcel of land in Tagaytay City. Petitioner accepted the
We held: offer subject to the condition that respondent Martels will settle their
obligation either by way of dacion en pago or through cash settlement
within a reasonable time. Thus, petitioner withdrew its complaint but
reserved its right to revive the same in the event that no settlement is arrived
The novation theory may perhaps apply prior to at. In December 2001, respondent Jose V. Martel wrote petitioner offering,
the filing of the criminal information in court by the state in lieu of the Tagaytay City property, computer-related services. Petitioner
prosecutors because up to that time the original trust filed with the Pasay City Prosecutors Office another complaint against
relation may be converted by the parties into an ordinary respondent Martels for SENCORs non-remittance of contributions.
creditor-debtor situation, thereby placing the complainant
in estoppel to insist on the original trust. But after the
justice authorities have taken cognizance of the crime and
ISSUE
instituted action in court, the offended party may no
longer divest the prosecution of its power to exact the Whether or not the concept of novation serves to abate the criminal liability
criminal liability, as distinguished from the civil. The of the respondents
crime being an offense against the state, only the latter
can renounce it.

It may be observed in this regard that novation is Part I, Case No. 70


not one of the means recognized by the Penal Code
whereby criminal liability can be extinguished; hence, the
role of novation may only be to either prevent the rise of BENJAMIN ("KOKOY") T. ROMUALDEZ, petitioner, vs.HON.
criminal liability or to cast doubt on the true nature of the SIMEON V. MARCELO, in his official capacity as the Ombudsman,
original basic transaction, whether or not it was such that and PRESIDENTIAL COMMISSION ON GOOD GOVERNMENT,
its breach would not give rise to penal responsibility, as respondents.
when money loaned is made to appear as a deposit, or
other similar disguise is resorted to. G.R. Nos. 165510-33, July 28, 2006

Intro/Doctrine/Ruling:

FACTS

respondent Martels are directors of SENCOR, an information technology Sec. 7 and 11 of RA 3019; Sec. 2 of Act No. 3326;
firm. Petitioner is a government-owned and controlled corporation
Article 91 of the RPC cannot be applied suppletorily to Act No. 3326
mandated by its charter, RA 1161, to provide financial benefits to private
because such Act is not a special law within the ambit of Article 10 of the
sector employees. Petitioner filed with the Pasay City Prosecutors Office a
RPC. petitioners cases in the Sandiganbayan to take declared invalid the
preliminary investigation conducted by the PCGG over the 24 offenses
ascribed to Romualdez.

In Romualdez v. Sandiganbayan, petitioner assailed the validity of the


1 Yes. Section 11 of RA No. 3019 provides that all offenses punishable
informations filed with the Sandiganbayan in Criminal Case Nos. 13406-
therein shall prescribe in 15 years. But prior to the amendment of Section 11
of R.A. No. 3019 by B.P. Blg. 195 which was approved on March 16, 1982, 13429 considering that the same were subscribed and filed by the PCGG. In
the prescriptive period for offenses punishable under the said statute was granting petitioners plea, this Court held, thus:
only ten (10) years. Thus, for offenses allegedly committed by the petitioner
from 1962 up to March 15, 1982, the same shall prescribe in 10 years. On
the other hand, for offenses allegedly committed by the petitioner during the Here, the informations were filed by an unauthorized party.
period from March 16, 1982 until 1985, the same shall prescribe in 15 The defect cannot be cured by conducting another
years. preliminary investigation. An invalid information is no
information at all and cannot be the basis for criminal
As to when these two periods begin to run, reference is made to Act No. proceedings.
3326 which governs the computation of prescription of offenses defined by
and penalized under special laws. Thus, this Court rules that the prescriptive
period of the offenses herein began to run from the discovery thereof or on Indeed, the nullity of the proceedings initiated by then Solicitor General
May 8, 1987, which is the date of the complaint filed by the former Chavez in 1987 with the PCGG and by the PCGG with the Sandiganbayan
Solicitor General Francisco I. Chavez against the petitioner with the PCGG. in 1989 is judicially settled. In contemplation of the law, no proceedings
exist that could have merited the suspension of the prescriptive periods.

Therefore, when the Office of the Special Prosecutor initiated the


preliminary investigation of Criminal Case Nos. 13406-13429 on March 3, 3 No. Section 2 of Act No. 3326 provides that the prescription shall begin to
2004 by requiring the petitioner to submit his counter-affidavit, the alleged run from the day of the commission of the violation of the law, and if the
offenses subject therein have already prescribed. same be not known at the time, from the discovery thereof and the
institution of judicial proceedings for its investigation and punishment. The
running of the prescriptive period shall be interrupted when proceedings are
instituted against the guilty person, and shall begin to run again if the
2 No. Under Section 2 of Act No. 3326, the prescriptive period shall be proceedings are dismissed for reasons not constituting jeopardy. Clearly,
interrupted when proceedings are instituted against the guilty person. Section 2 of Act No. 3326 did not provide that the absence of the accused
However, there is no such proceeding instituted against the petitioner to from the Philippines prevents the running of the prescriptive period. Thus,
warrant the tolling of the prescriptive periods of the offenses charged the only inference that can be gathered from the foregoing is that the
against him. In Romualdez v. Sandiganbayan, petitioner averred that PCGG legislature, in enacting Act No. 3326, did not consider the absence of the
acted without jurisdiction and/or grave abuse of discretion in conducting a accused from the Philippines as a hindrance to the running of the
preliminary investigation of cases not falling within its competence. This prescriptive period. Expressio unius est exclusio alterius.
Court, in its resolve to deal with the merits of the case to remove the
possibility of any misunderstanding as to the course which it wishes
Also, while it is true that Article 10 of the Revised Penal Code makes the 3019?
Code suppletory to special laws, Act No. 3326 cannot fall within the ambit
of special law as contemplated and used in Article 10 of the RPC because it
Case #72.
does not define any acts which are punishable and provides penalties for
them. Special law is defined as penal laws that punish acts not defined and People of the Philippines vs Vergara
penalized by the Penal Code of the Philippines.
GR No 177763; July 3, 2013
Thus, Art. 91 cannot apply suppletorily to Act No. 3326.
Doctrines:
Facts: Petitioner is being charged with violations of Section 7 of RA No.
3019 for failure to file his Statements of Assets and Liabilities for the period Self-Defense, elements: (1) unlawful aggression on the part of the victim;
1967-1985 during his tenure as Ambassador Extraordinary and (2) reasonable necessity of the means employed to prevent or repel such
Plenipotentiary and for the period 1963-1966 during his tenure as Technical aggression; and (3) lack of sufficient provocation on the part of the person
Assistant in the DFA. He claims that the court should dismiss the criminal resorting to self-defense.
cases pending against him on the ground of prescription.
A person who invokes self-defense has the burden of proof. He must prove
all the elements of self-defense. However, the most important of all the
elements is unlawful aggression on the part of the victim.
The Ombudsman and the PCGG argue that that the filing of the complaint
with the Presidential Commission on Good Government (PCGG) in 1987 Unlawful aggression must be proved first in order for self-defense to be
and the filing of the information with the Sandiganbayan in 1989 successfully pleaded, whether complete or incomplete. Unlawful aggression
interrupted the prescriptive period; that the absence of the petitioner from is an actual physical assault, or at least a threat to inflict real imminent
the Philippines from 1986 until 2000 also interrupted the aforesaid period injury, upon a person. In case of threat, it must be offensive and strong,
based on Article 91 of the Revised Penal Code; that considering that both positively showing the wrongful intent to cause injury. It "presupposes
RA No. 3019 and Act No. 3326 or the Act To Establish Periods of actual, sudden, unexpected or imminent danger - not merely threatening and
Prescription For Violations Penalized By Special Acts and Municipal intimidating action." It is present "only when the one attacked faces real and
Ordinances and to Provide When Prescription Shall Begin To Run, are immediate threat to ones life."
silent as to whether prescription should begin to run when the offender is
Treachery, how manifested: the offender commits any of the crimes
absent from the Philippines, the Revised Penal Code, which answers the
against persons, employing means, methods, or forms in the execution,
same in the negative, should be applied.
which tend directly and specially to insure its execution, without risk to the
offender arising from the defense which the offended party might make.

Issues: 1. Whether the offenses for which petitioner are being charged have Article 248. Murder. - Any person who, not falling within the provisions of
already prescribed? Article 246, shall kill another, shall be guilty of murder and shall be
punished by reclusion perpetua to death if committed with any of the
2 WON the filing of the complaint with the PCGG and Sandiganbayan following attendant circumstances:
interrupted the running of the prescriptive period?
3 WON Article 91 of the RPC applies suppletorily to Act No. 3326, such that
it suspended the running of the prescriptive period of 10 or 15 years in RA
1) With treachery, taking advantage of superior strength, with the aid of was totally unaware of the evil that would befall him. The number and
armed men, or employing means to weaken the defense or of means or severity of the wounds received by the victim indicated that he was
persons to insure or afford impunity. rendered immobile and without any real opportunity to defend himself other
than feebly raising his arm to ward off the attack.

Credibility of Witnesses:
DAYAP VS SENDIONG
When it comes to the matter of credibility of a witness, settled are the
guiding rules some of which are that (1) the Appellate court will not disturb Article 365 of the Revised Penal Code punishes any person who, by
the factual findings of the lower Court, unless there is a showing that it had reckless imprudence, commits any act which, had it been intentional, would
overlooked, misunderstood or misapplied some fact or circumstance of constitute a grave felony, with the penalty of arresto mayor in its maximum
weight and substance that would have affected the result of the case, which period to prision correccional in its medium period. When such reckless
showing is absent herein; (2) the findings of the Trial Court pertaining to the imprudence the use of a motor vehicle, resulting in the death of a person
credibility of a witness is entitled to great respect since it had the attended the same article imposes upon the defendant the penalty of prision
opportunity to examine his demeanor as he testified on the witness stand, correccional in its medium and maximum periods;
and, therefore, can discern if such witness is telling the truth or not; and (3)
a witness who testifies in a categorical, straightforward, spontaneous and Where a reckless, imprudent, or negligent act results in two or more
frank manner and remains consistent on cross-examination is a credible grave or less grave felonies, a complex crime is committed. Article 48 of
the Revised Penal Code provides that when the single act constitutes two or
witness.
more grave or less grave felonies, or when an offense is a necessary means
for committing the other, the penalty for the most serious crime shall be
imposed, the same to be applied in its maximum period. Since Article 48
Facts: Accused Vergara and Inocencio were charged of murder qualified by speaks of felonies, it is applicable to crimes through negligence in view of
treachery. Victim was allegedly stabbed by the accused while the latter were the definition of felonies in Article 3 as acts or omissions punishable by
causing some disturbance in the street. Exchange of words lead to the law committed either by means of deceit (dolo) or fault (culpa). Thus, the
incident initiated by the accused and the retort of the victim was not likened penalty imposable upon petitioner, were he to be found guilty, is prision
by the former. correccional in its medium period (2 years, 4 months and 1 day to 4 years)
and maximum period (4 years, 2 months and 1 day to 6 years).
Issues:
FACTS:
1. Whether or not the accused had basis to interpose self-defense? No, Dayap allegedly drove in a reckless manner a 10-wheeler cargo truck hitting
the accused had no basis to interpose self-defense. Elements are not all an automobile driven by Sendiong who was with two female passengers.
present. In fact, the unlawful aggression came from the accused and not the Such incident caused the death of Sendiong, less serious physical injuries
victim. The victim was passing in peace when they approached him and on the bodies of the two female passengers and extensive damage to the
initiated the act of killing causing the life of the victim in peril. automobile. Hence, Dayap was charged with the crime of Reckless
Imprudence resulting to Homicide, Less Serious Physical Injuries, and
2. Whether or not treachery is present? Yes, treachery is present. Here, Damage to Property.
accused-appellant Vergara after exchanging words with the victim, threw
his arm around the victims shoulder and proceeded to stab him. The victim ISSUE:
WON the RTC has jurisdiction to hear a criminal case involving complex They then agreed to conduct the pay off in the morning of April 7, 2001 at
crimes such as reckless imprudence resulting in homicide, less serious Pitangs Carinderia in Kapatagan, Lanao del Norte.
physical injuries and damage to property?

RULING:
NO. When this case was filed on 29 December 2004, Section 32(2) of Batas Issue:
Pambansa Bilang 129 had already been amended by R.A. No. 7691. R.A. Whether the two accused are guilty of violating the crime of Kidnapping
No. 7691 extended the jurisdiction of the first-level courts over criminal for Ransom under Article 267 ofthe RPC, as amended by RA No. 7659?
cases to include all offenses punishable with imprisonment not exceeding
six (6) years irrespective of the amount of fine, and regardless of other
imposable accessory or other penalties including those for civil liability. It
explicitly states "that in offenses involving damage to property through Ruling: Yes
criminal negligence, they shall have exclusive original jurisdiction thereof."
It follows that criminal cases for reckless imprudence punishable Kidnapping; Elements.The crime has the following elements: (1) the
with prision correccional in its medium and maximum periods should
fall within the jurisdiction of the MTC and not the RTC. Clearly, offender is a private individual; not either of the parents of the victim or a
therefore, jurisdiction to hear and try the same pertained to the MTC and the public officer who has a duty under the law to detain a person; (2) he
RTC did not have original jurisdiction over the criminal case. Consequently,
the MTC of Sibulan, Negros Oriental had properly taken cognizance of the kidnaps or detains another, or in any manner deprives the latter of his
case and the proceedings before it were valid and legal.
liberty; (3) the act of detention or kidnapping must be illegal and (4) in the
commission of the offense, any of the following circumstances is present:
People vs Mamantak
(a) the kidnapping or detention lasts for more than three days; (b) it is
GR no. 174659
committed by simulating public authority; (c) any serious physical injuries
July 28, 2008
are inflicted upon the person kidnapped or detained or threats to kill him are
made or (d) the person kidnapped or detained is a minor, female or a public
Article 267 of the RPC official.

The essence of the crime of kidnapping is the actual deprivation of the


Facts:
victims liberty coupled with the intent of the accused to effect it; If the
victim is a minor, or the victim is kidnapped and illegally detained for the
Christopher, a two-year old minor, had disappeared from mcdonalds in purpose of extorting ransom, the duration of his detention becomes
Binondo when his mom and his sister were ordering from the counter. On
February 25, 2001, Teresa received a call from a woman and claimed to inconsequential.
have custody of Christopher and asked forP30,000 in exchange for the boy.
SAFEGUARD SECURITY vs. TANGCO 0-97-73806 and assigned to Branch 78. Respondents reserved their right to
file a separate civil action in the said criminal case. The RTC of Quezon
GR No. 165732, December 14, 2006 City subsequently convicted Pajarillo of Homicide in its Decision dated
Austria Martinez, J.: January 19, 2000. On appeal to the CA, the RTC decision was affirmed with
modification as to the penalty in a Decision.

Meanwhile, on January 14, 1998, respondents filed with RTC, Branch 273,
DOCTRINE: An act or omission causing damage to another may give rise Marikina City, a complaint5 for damages against Pajarillo for negligently
to two separate civil liabilities on the part of the offender, i.e., (1) civil shooting Evangeline and against Safeguard for failing to observe the
liability ex delicto, under Article 100 of the Revised Penal Code; and (2) diligence of a good father of a family to prevent the damage committed by
independent civil liabilities, such as those (a) not arising from an act or its security guard. Respondents prayed for actual, moral and exemplary
omission complained of as a felony, e.g., culpa contractual or obligations damages and attorney's fees.
arising from law under Article 31 of the Civil Code, intentional torts under
Articles 32 and 34, and culpa aquiliana under Article 2176 of the Civil In their Answer,6 petitioners denied the material allegations in the
Code; or (b) where the injured party is granted a right to file an action complaint and alleged that Safeguard exercised the diligence of a good
independent and distinct from the criminal action under Article 33 of the father of a family in the selection and supervision of Pajarillo; that
Civil Code. Either of these liabilities may be enforced against the offender Evangeline's death was not due to Pajarillo's negligence as the latter acted
subject to the caveat under Article 2177 of the Civil Code that the offended only in self-defense.
party cannot recover damages twice for the same act or omission or under The RTC found respondents to be entitled to damages. It rejected Pajarillo's
both causes. claim that he merely acted in self-defense. It gave no credence to Pajarillo's
Provisions mentioned: Art. 2176, 2177 of the Civil code, Article 103 of bare claim that Evangeline was seen roaming around the area prior to the
the RPC shooting incident since Pajarillo had not made such report to the head office
and the police authorities. The RTC further ruled that being the guard on
FACTS: duty, the situation demanded that he should have exercised proper prudence
and necessary care by asking Evangeline for him to ascertain the matter
On November 3, 1997, at about 2:50 p.m., Evangeline Tangco (Evangeline) instead of shooting her instantly; that Pajarillo had already been convicted
went to Ecology Bank, Katipunan Branch, Quezon City, to renew her time of Homicide in Criminal Case No. 0-97-73806; and that he also failed to
deposit per advise of the bank's cashier as she would sign a specimen card. proffer proof negating liability in the instant case.
Evangeline, a duly licensed firearm holder with corresponding permit to
carry the same outside her residence, approached security guard Pajarillo,
who was stationed outside the bank, and pulled out her firearm from her bag
to deposit the same for safekeeping. Suddenly, Pajarillo shot Evangeline The RTC also found Safeguard as employer of Pajarillo to be jointly and
with his service shotgun hitting her in the abdomen instantly causing her severally liable with Pajarillo. It ruled that while it may be conceded that
death. Safeguard had perhaps exercised care in the selection of its employees,
particularly of Pajarillo, there was no sufficient evidence to show that
Lauro Tangco, Evangeline's husband, together with his six minor children Safeguard exercised the diligence of a good father of a family in the
(respondents) filed with the Regional Trial Court (RTC) of Quezon City, a supervision of its employee; that Safeguard's evidence simply showed that it
criminal case of Homicide against Pajarillo, docketed as Criminal Case No. required its guards to attend trainings and seminars which is not the
supervision contemplated under the law; that supervision includes not only YES. Safeguard insists that the claim for damages by respondents is based
the issuance of regulations and instructions designed for the protection of on culpa aquiliana under Article 217611 of the Civil Code, in which case, its
persons and property, for the guidance of their servants and employees, but liability is jointly and severally with Pajarillo. However, since it has
also the duty to see to it that such regulations and instructions are faithfully established that it had exercised due diligence in the selection and
complied with. supervision of Pajarillo, it should be exonerated from civil liability.

Petitioners appealed the RTC decision to the CA. On July 16, 2004, the CA The law at the time the complaint for damages was filed is Rule 111 of the
issued its assailed Decision, the dispositive portion of which reads: 1985 Rules on Criminal Procedure, as amended, to wit:

IN VIEW OF ALL THE FOREGOING, the appealed decision is hereby


AFFIRMED, with the modification that Safeguard Security Agency, Inc.'s
civil liability in this case is only subsidiary under Art. 103 of the Revised SECTION 1. Institution of criminal and civil actions. - When a
Penal Code. criminal action is instituted, the civil action for the recovery of
civil liability is impliedly instituted with the criminal action,
In finding that Safeguard is only subsidiarily liable, the CA held that the unless the offended party waives the civil action, reserves his
applicable provisions are not Article 2180 in relation to Article 2176 of the right to institute it separately, or institutes the civil action prior to
Civil Code, on quasi-delicts, but the provisions on civil liability arising the criminal action.
from felonies under the Revised Penal Code; that since Pajarillo had been
found guilty of Homicide in a final and executory judgment and is said to Such civil action includes recovery of indemnity under the
be serving sentence in Muntinlupa, he must be adjudged civilly liable under Revised Penal Code, and damages under Articles 32, 33, 34, and
the provisions of Article 100 of the Revised Penal Code since the civil 2176 of the Civil Code of the Philippines arising from the same
liability recoverable in the criminal action is one solely dependent upon act or omission of the accused.
conviction, because said liability arises from the offense charged and no Respondents reserved the right to file a separate civil action and in fact filed
other; that this is also the civil liability that is deemed extinguished with the the same on January 14, 1998.
extinction of the penal liability with a pronouncement that the fact from
which the civil action might proceed does not exist; that unlike in civil The CA found that the source of damages in the instant case must be the
liability arising from quasi-delict, the defense of diligence of a good father crime of homicide, for which he had already been found guilty of and
of a family in the employment and supervision of employees is inapplicable serving sentence thereof, thus must be governed by the Revised Penal Code
and irrelevant in civil liabilities based on crimes or ex-delicto; that Article
103 of the Revised Penal Code provides that the liability of an employer for An act or omission causing damage to another may give rise to two separate
the civil liability of their employees is only subsidiary, not joint or solidary. civil liabilities on the part of the offender, i.e., (1) civil liability ex delicto,
under Article 100 of the Revised Penal Code; and (2) independent civil
ISSUE/S: liabilities, such as those (a) not arising from an act or omission complained
of as a felony, e.g., culpa contractual or obligations arising from law under
Whether or not the CA erred in ruling that the liability of Safeguard Article 31 of the Civil Code, intentional torts under Articles 32 and 34, and
Security is subsidiary culpa aquiliana under Article 2176 of the Civil Code; or (b) where the
HELD: injured party is granted a right to file an action independent and distinct
from the criminal action under Article 33 of the Civil Code. Either of these
liabilities may be enforced against the offender subject to the caveat under
Article 2177 of the Civil Code that the offended party cannot recover G.R. No. 178115 July 28, 2014
damages twice for the same act or omission or under both causes.
PEOPLE OF THE PHILIPPINES, Plaintiff-Appellee,
A reading of respondents' complaint shows that the latter are invoking their
vs.
right to recover damages against Safeguard for their vicarious responsibility
JOJO SUMILHIG, RICARDO SUMILHIG alias CARDING
for the injury caused by Pajarillo's act of shooting and killing Evangeline
SUMILHIG, PASOT SALOLI, ERIC ENOC, WARLITO
under Article 2176, Civil Code. The scope of Article 2176 is not limited to
MONTEBON,* and CIO LIMAMA, Accused,
acts or omissions resulting from negligence. Well-entrenched is the doctrine
that Article 2176 covers not only acts committed with negligence, but also
acts which are voluntary and intentional. JOJO SUMILHIG, RICARDO SUMILHIG alias CARDING
SUMILHIG, and PASOT SALOLI, Accused-Appellants.
In Elcano vs Hill, the court ruled that: Article 2176, where it refers to "fault
or negligence," covers not only acts "not punishable by law" but also acts
criminal in character, whether intentional and voluntary or negligent.
Consequently, a separate civil action lies against the offender in a criminal FACTS: Appellants, together with the accused Eric Enoc, Warlito Montibon
act, whether or not he is criminally prosecuted and found guilty or and Cio Limama, were charged with double murder and double frustrated
acquitted, provided that the offended party is not allowed, if he is actually murder in an Amended Information,3 the accusatory portion of which reads:
charged also criminally, to recover damages on both scores, and would be
entitled in such eventuality only to the bigger award of the two, assuming That on or about October 31, 1998, inthe Municipality of Kiblawan,
the awards made in the two cases vary. In other words, the extinction of Province of Davao del Sur, Philippines, and within the jurisdiction of this
civil liability referred to in Par. (e) of Section 3, Rule 111, refers exclusively Honorable Court, the above-named accused, conspiring, confederating and
to civil liability founded on Article 100 of the Revised Penal Code, whereas helping one another, armed with assortedfirearms, with intent to kill with
the civil liability for the same act considered as quasi-delict only and not as treachery and evident premeditation, did, then and there willfully,
a crime is not extinguished even by a declaration in the criminal case that unlawfully and feloniously, simultaneously strafe the house of Eugenio
the criminal act charged has not happened or has not been committed by the Santander resulting to death of [Cresjoy] Santander and RollySantander and
accused. Briefly stated, We here hold, in reiteration of Garcia, that culpa seriously wounding Marissa Santander and Micel Santander, which would
aquiliana includes voluntary and negligent acts which may be punishable by have caused their death had there been no timely and able medical
law. assistance rendered to them, to the damage and prejudice of the offended
The civil action filed by respondents was not derived from the criminal parties.
liability of Pajarillo in the criminal case but one based on culpa aquiliana or
quasi-delict which is separate and distinct from the civil liability arising
from crime. The source of the obligation sought to be enforced in the civil
case is a quasi-delict not an act or omission punishable by law. ISSUE:

Since the civil liability was founded on a quasi-delict and not from the
WON: the crime committed is a complex crime
crime of homicide. The liability of Safeguard Security must be Solidary and
not Subsidiary.
Rulings: c. Frustrated Murder for the shooting of Marissa Santander and x x
x are sentenced to suffer an imprisonment ofsix (6) years, four (4)
Ruling of the Regional Trial Court months and [ten] (10) days of prision mayoras minimum to
fourteen (14) years, eight (8) months and twenty (20) days of
The RTC convicted the appellants ofthe complex crime of double murder reclusion temporalas maximum; d. Frustrated Murder for the
and double frustrated murder and sentenced them to suffer the penalty of shooting of Micel Santander and x x x are sentenced to suffer an
death. It further ordered them to indemnify, jointlyand severally, the heirs of imprisonment ofsix (6) years, four (4) months and ten (10) days of
Cresjoy and Rolly the sum of P100,000.00 as civil indemnity, and the prision mayoras minimum to fourteen (14) years, eight (8) months
surviving victims, Marissa and Micel, the sums of P50,000.00 and twenty (20) days of reclusion temporalas maximum. All
and P30,000.00 as moral and exemplary damages, respectively.8 accused are ordered to indemnify jointly and severally the heirs of
Crisjoy Santander and Rolly Santander the sum of P100,000.00
and the surviving victims Marissa Santander and Micel
Ruling of the Court of Appeals
Santander P50,000.00 as moral damages and P30,000.00 as
exemplary damages, without subsidiary imprisonment in case of
On appeal, the CA did not find any reason to disturb the findings of the
insolvency.
RTC. However, it found merit in appellants argument that the crime
committed could not have been a complex crime since the death and
Ruling of the Supreme Court
injuries complained of did not result from a single act but from several and
distinct acts of shooting. And as treachery was alleged in the Amended
Information and sufficiently proven during trial, appellants should be The appeal has no merit.
convicted instead of two counts of murder and two counts of frustrated Appellants conviction was based on
murder. Thus, in rendering its Decision9 dated July 28, 2006, the CA their positive identification by the
disposed ofthe case as follows: prosecution witnesses.

WHEREFORE, the judgment of conviction of appellants Jojo Sumilhig, True, the RTC noted in its Decision the existence of motive on the part of
Alias Carding Sumilhig and Pasot Saloli is affirmed butmodified as follows Jojo for committing the crime as well as Pasots incredulous claim of
ignorance on almost about everything. It is well to note, however, that the
said court neither based the appellants conviction on the existence of such
motive nor on Pasots weak defense of ignorance alone, but upon the
Appellants Juan "Jojo" Sumilhig, Alias Carding Sumilhig and Pasot Saloli,
prosecution witnesses identification of appellants as the assailants, viz:
are found guilty beyondreasonable doubt of:

Assessing the evidence presented by both [P]rosecution and defense, we see


a. Murder for killing Crisjoy Santander, and x x x are sentenced to
a less than glaring hint of vendetta. As part of his defense, the accused Jojo
suffer the penalty of reclusion perpetua;
Sumilhig narrated that his family was massacred by Jerry Santander, brother
of Remigio Santander [in] February 1998. Short of admitting the crime,
b. Murder for the killing of Rolly Santander, and x x x are
Sumilhig stated that because of this, he harbored ill feelings not only against
sentenced to suffer the penalty of reclusion perpetua;
Jerry and Carlos Santander but also against their family. Thus a clear motive
for killing the Santander family has been established giving credence to
prosecution witnesses allegation that after the strafing Jojo Sumilhig All these observations however become insignificant in the face of the
shouted"Nakabalos na ko!" The likelihood of his intention to wipe out the positive and spontaneous identification of the assailants/accused by credible
said family became even more apparent. witnesses Jerry Masaglang and Remigio Santander.15

Despite his positive assertion that it was the Santanders that killed his There is no reason to doubt Jerry and Marios identification of the
family, he did not file any case against them. It was only after he was appellants considering that (1) Jerry was just six meters away from
arrested that he filed a complaint against Jerry and Carlos Santander. them;16 (2) the moon was bright and Jerry was familiar with all the accused
as most of them are his relatives;17 and, (3) Mario knows Jojoever since he
His alibi likewise failed to meet the stringent requisites of the Rules. Even was small.18 Besides, "[t]ime-tested is the rulethat between the positive
as Dr. Quirapas appeared determined to rule out the possibility that he could assertions of prosecution witnesses and the negative averments of the
walk without crutches five months after his discharge, the same was based accused, the former undisputedly [deserve] more credence and [are] entitled
on general medical prognosis. Such prognostication admits certain to greater evidentiary weight."19
exception[s], as could be gleaned from the testimony of the doctor himself
that the healing period may vary depending on the age and physical Anent the respective alibis interposed by appellants, suffice it to say that
condition of the patient. Notably Jojo Sumilhig was then 23 years old. "[a]libi cannot prevail over the positive identification of a credible
witness."20
What was certain was the positive identification made by Jerry Masaglang
and Remegio Santander of all of the accused. There was conspiracy among the accused.

The "overkill" by which the accused Pasot Saloli and Carding Sumilhig "[C]onspiracy exists when two or more persons come to an agreement
claimed total ignorance of almost anything only served to arouse concerning the commission of a felony and decide to commit it."21 It is not
incredulity. Both accused claimed they could not read, write, tell time, day, necessary to adduce evidence of a previous agreement to commit a
month or year. Neither could [they] allegedly speak [or] understand crime.22"Conspiracy may be shown through circumstantial evidence,
Visayan, which is of common knowledge to be widely spoken in almost deduced from the mode and manner in which the offense was perpetrated,
every part of Mindanao. Saloli claimed he did not know what day [it] was or inferred from the acts of the accused themselves when such leadto a joint
when he was testifying, or the day before and after that. Both claimed they purpose and design, concerted action, and community of interest."23
did not know the complainants or of the massacre that took place.
Here, there is no proof of a previous agreement among the accused but there
xxxx is a series of events that clearly established conspiracy among them. First,
they were all armed with firearms. Second, they surreptitiously approached
More importantly, these claims [of] utter ignorance are belied by the the crime scene. Third, when they were within close range of the intended
evasiveness by which all three accused answered in obvious effort to avoid victims, they simultaneously discharged their firearms. Fourth, they ceased
criminal responsibility. Behind the faade of ignorance and lack of firing at the same time and fled together. Undoubtedly, their acts before,
education lurks a calculating mind. We find [it] difficult to ascribe during and immediately after strafing the house of Eugenio evince their
innocence to the accused when traces of ingenuity and craftiness unanimity in design, intent and execution.24 Treachery attended the
characterize their testimonies. commission of the crime.
"There is treachery whenthe offender commits any of the crimes against the Parricide - other than the fact of killing - is the relationship of the offender
person, employing means, methods orforms in the execution thereof which to the victim.23 All the elements are present in this case. Jose, the victim,
tend directly and specially to insure the execution, without risk to himself was killed by accused-appellant, his own son.
arising from [any] defense which the offended party might make."25
The crime of Parricide is punishable by the indivisible penalties
of reclusion perpetua to death. With one mitigating circumstance, namely,
Treachery is evident in this case as the suddenness and unexpectedness of
voluntary surrender, and no aggravating circumstance, the imposition of the
the assault deprived the victims of an opportunity to resist it or offer any lesser penalty of reclusion perpetua on accused-appellant was proper.
defense of their persons. This is considering that the victims were unaware
that they would be attacked by appellants with a hailof bullets from their FACTS:
firearms fired at close range. Indeed, "[t]he suddenness of the attack, The said accused, did then and there willfully, unlawfully and feloniously,
without the slightest forewarning thereof, placed the [victims] x x x in such and with evident premeditation, that is, having conceived and deliberated to
a position that they could not have defended themselvesfrom the aggression kill his own father Jose Guting y Ibarra, 67 years old, married, while inside
x x x."26 their residential house, and armed with a bladed weapon, suddenly and
unexpectedly stabbed several times the victim, employing means, manner
The crime committed is two counts of and form in the execution thereof which tender directly and specially to
murder and two counts of frustrated insure its commission without danger to the person of said accused, the
murder. result of which attack was that said victim received multiple stab wounds on
his body which directly caused his instantaneous death.
PEOPLE OF THE PHILIPPINES, Plaintiff-Appellee, v. ADRIAN The RTC promulgated its Decision on June 24, 2010 finding accused-
GUTING Y TOMAS, Accused-Appellant. appellant guilty of Parricide based on his verbal admission that he killed his
G.R. No. 205412, September 09, 2015 LEONARDO-DE CASTRO, J. father, Jose. Even assuming that accused-appellant's admission was
inadmissible in evidence, the RTC adjudged that the prosecution was still
DOCTRINE: able to establish sufficient circumstantial evidence which, taken
Article 246 of the Revised Penal Code defines Parricide as follows: collectively, pointed to accused-appellant as the perpetrator of the brutal
killing of his father.
Art. 246. Parricide. - Any person who shall kill his father, mother, or child,
whether legitimate or illegitimate, or any of his ascendants, or descendants, ISSUE:
or his spouse, shall be guilty of parricide and shall be punished by the Whether the accused-appellant guilty beyond reasonable doubt of the crime
penalty of reclusion perpetua to death. charged despite the prosecution's failure to overthrow the constitutional
presumption of innocence in his favor.
Parricide is committed when: (1) a person is killed; (2) the deceased is
killed by the accused; and (3) the deceased is the father, mother, or child,
whether legitimate or illegitimate, or a legitimate other ascendant or other
descendant, or the legitimate spouse of the accused. The key element in

Вам также может понравиться